You are on page 1of 514
ATEXTBOOK OF ———— a ‘ical cs 3 Published by : LAXMI PUBLICATIONS (P) LTD 113, Golden House, Daryaganj, New Delhi-110002 Phone : 011-43 53 25 00 Fax : 011-43 53 25 28 info@laxmipublications.com Compiled by : Smt. Ramesh Rajput © All rights reserved with the Author and Publishers. No part of this publication may be reproduced, stored in a retrieval system, or transmitted in any form or by any means, electronic, mechanical, photocopying, recording or otherwise without the prior written permission of the publisher: Price : Rs, 195.00 Only. First Edition : 2004 Reprint : 2006 Offices : India USA ® Bangalore (Phone : 080-26 61 15 61) * Boston # Chennai (Phone : 044-24 34 47 26) 11, Leavitt Street, Hingham, @ Cochin (Phone ; 0484-239 70 04) MA 02043, USA e Guwahati (Phones : 0361-254 36 69, 251 38 81) Phone : 781-740-4487 « Hyderabad (Phone : 040-24 75 02 47) « Jalandhar City (Phone : 0181-222 12 72) @ Kolkata (Phones : 033-22 27 97 73, 22 27 52 47) « Lucknow (Phone : 0522-220 95 78) « Mumbai (Phones : 022-24 91 64 15, 24 92 78 69) @ Ranchi (Phone : 0651-230 77 64) EET-0622-196-ELECTRICAL TECHNOLOGY (HR) C—13204/08/10- ‘Typeset of : Goswami Printers, Delhi. Printed at ; Mehra Offset Press, New Delhi. CONTENTS Chapters Pep p BREEBRPRPREE Syllabus D.C. CIRCUITS AND NETWORK ANALYSIS: Definitions of Important Terms Limitati [Ohms L Kirchhoff’s Laws Applications of Kirchhoff's Laws 4.1. _Branch-current method 4.2, Maxwell's loop (or mesh) current method 4.3, Nodal voltage method Solving Equations by Determinants—Cramer’s Rule ‘Superposition Theorem Thevenin’s Ti Norton's Theorem Maximum Power Transfer Theorem Delta Star Transformation Compensation Theorem Reciprocity Theorem Millman’s Theorem Highlights Objective Type Questions i stions Theoretical Questions Unsolved Examples PPPr bp AC, CIRCUITS Introduction to Alternating Current Generation and tions of Alternat Volt and Currents Alternating Voltage and Current Single-phase Circuits 4.1. A.C, through pure ohmic resistance alone 4.2. A.C. through pure inductance alone 43. AC. throu; Mare Cal alone 44, Phasor algebra 4.6. A.C. parallel circuits ‘Transients 5.1, _ General aspects (vi) E “ccmecl REBEEERRBueaaReeBE (a) F & a & THREE-PHASE A.C. NETWORK Introduction Advantages of Polyphase Systems Generation of Three-phase Voltages: Phase Sequence and Numbering of Phases Inter-connection of Three Phases ‘Star or Wye (¥) Connection ‘Delta (A) or Mesh Connection Comparison between Star and Delta Systems Measurement of Power in 3- circuit 9.1.__Three-wattmeters method 9.2. _Two-wattmetors method 9.2. _One-wattmeter method Measurement of Reactive Volt Amperes types of Energy Meters 11.1.___Motor meters 1L.2__Motor-driven meter—watt-hour meter 11.3. __Induction type watt-hour meter we 11.4.__Induetion single-phase energy meters a Power Factor Improvement Earthing and Grounding 13.1. General aspects 13.2. Objects of earthing 13.3.__ Specifications required for earthing as per 1.8.1. 13.4. Methods of earthing 13.5. Sizes of earth wire and earth plate for domestic and installati 13.6.__Indian electricity rules 13.7, Measurement of earth resistance by earth tester 13.8.__Earthing of a power system Highlights Objective stions Theoretical Questions Unsolved Examples PRENPPEE RE p FE EB BEBREEEB seunaeseseerncexeesnesss 4. TRANSFORMERS General Aspects “ Rete Defiel Working Principle of a Transformer ‘Transformer Ratings Kinds.of Transformers ‘Transformer Construction 6.1, _Core-type transformer 6.2. Shell type transformer os 63. Spiral core transformer ~ ‘Transformer Windings. Terminals. Tappings and Bushings - 7.1. Transformer windings 12___Terminals and leads PRB pe n 7.3. _Tappings 74. Bushings ses eueenneeee | Chapters & Transformer Cooling 8.1. Cooling methods &.2___Transformer oi) 8.3. Conservator and breather a 9.1. Elementary theory of an ideal transformer 9.2. E.M_F. equation of a transformer 9.3. Voltage transformation ratio (K) 9.4. Transformer with losses but no magnetic leakage 9.5. Resistance and magnetic leakage 9.6. Transformer with resistance and leakage reactance 9.7. Equivalent resistance and reactance 9.8. Total voltage drop in a transformer 9.9. Equivalent circuit 9.10 Transformer Tests 9.11. Regulation of a transformer 9.12. Percentage resistance and reactance 9.13. Transformer lasses, 9.14. Transformer efficiency 9.15. All-day efficiency 9.16. Transformer noise 9.17 Auto-transformer 9.18. Polarity of transformers ‘Three phase transformer 10.1. _Three-phase transformer connections 10.2. _Three-phase transformer construction 10.3. Parallel operation of 3-phase transformers Highlights Objective T) uestions Theoretical Questions Unsolved Examples F ROTATING MACHINES Direct C D.C.) Machi 1.1___Construction of D.C, machines 1.2. _E.mf. equation of a generator 13. Types of D.C. generators 1.4. Parallel operation of D.C. generators 14____Dirert current motor Synchronous Machines 2.1. Synchronous gencrator or alternator 2.2. Synchronous motor Polyphase Induction Motor 3.1. General aspects 3.2 Classification of AC. c sonal detai 3.4. Production of rotating magnetic field 3.5. Theory of operation of an induction motor 3.6. Slip 3.7. Frequency of rotor current 4.8. Rotor emf and rotor current ceteeeeereeveenses RERRERBEBREEEEERERRERBBEEEREERRE ( xiii) Chapters Pe Prep 3.9. Torque and power 3.10. Effect of change in supply voltage on starting torque 3.11. Effect of change in supply voltage on torque and slip 3.12. Full-load torque and maximum torque 2.13. Starting torque and maximum torque 3.14. ue-slip and Wid curves 3.15. Operating characteristics of a 3-phase squirrel-cage induction motor 3.16. Operating characteristics of a wound-rotor (slip ring) induction motor 3.17. Starting of induction motors 3.18. Squirrel-cage motors—advantages, disadvantages and applications 3.19. Wound rotor (or slip ring) induction motors—advantages, disadvantages and applications 3.20. Comparison of a squirrel-cage and a slip ring (or phase wound) induction motors Single Phase Motors 4.1. General aspects 4.2. Types of single-phase motor 4.3. Single-phase induction motors 44. Split-phase motors 45. Single-phase commutator motors 4.6. Single-phase synchronous motors Insulating Materials Rating and Heating of D.C. Machines 6.1, Rating 6.2 Heating 6.3. Selection of D.C. generators and motors 6.4. Types of D.C. machine enclosures Rating Specifications of A.C. Machines Duty les Cooling of Electrical Machines Highlights—D.C. Machines Objective Type Questions Theoretical Questions Unsolved Examples Highlights—Synchronous Machines Objective lwestions Theoretical Questions Unsolved Examples Highlights—Polyphase Induction Motor Objective Type Questions Theoretical Questions Unsolved Examples Highlights—Single Phase Motors Objective Type Questions Theoretical Questions sg 3 BEEBSRERES22 B BRE 413 413 EE BBE 424 BES 432 EREEBREB Pe 2. 3. MEASURING INSTRUMENTS Introdueti Classificati Electrical Principles of Operation Electrical Indicating Instruments .1.___Essential features seep (xiv) Chapters: ie PP 3.2. Deflecting device 3.3. Controlling devices 3.4, Damping devices Moving-Iron Instruments (Ammeters and Voltmeters) 4.1, Attraction type 4.2. Repulsion type 4.3. Advantages and disadvantages of moving-iron instruments 44. Sources of errors Moving-Coil Instruments 5.1, Permanent-magnet moving-coil type (PMMC) instruments 6.2. Electrodynamic or dynamometer instruments Rectifier Instruments Wattmeters. 7.1.__ Dynamometer wattmeter 7.2.__Induction wattmeters Integrating Meters (Energy Meters) 8.1. Essential characteristics of energy meters 8.2. Types of energy meters 8.3, Motor meters: 8.4. Moter-driven meter—watt-hour meter 8.5. _Induction type watt-hour meter Measurement of Resistance &1._Voltmeter-ammeter method 9.2. Substitution method 9.3. Measurement of resistance by the wheatstone bridge ‘The Potentiometer Meggar Instrument Transformers 12.1__Patential transformers (P.T.} 12.2.__ Current transformers (C.T.) Highlights Theoretical Questions Unsolved Examples eae 457 457 RBSBBRREBEBE 476 476 476 BERRESSEBEBBS MISCELLANY a istics of DCO 1.1. Separately excited generator 1.2. Building up the voltage of self-excited shunt generator 1.3. Shunt generator characteristies 14. __ Series generator 1.5. Compound wound generator 1.6. Applications of D.C. generators Speed Control of D.C. Motors 21, Factors controlling the speed 2.2. Field control method 22. Rheostatic control 24. Voltage control Electromechanical Energy Conversion 8.1__Introduction 3.2. Principle of energy conversion 3.3. Faraday’s laws of electromagnetic induction ceeceansesneanel s 8 (xv) Chapters Pages 3.4. Singly and multiply-excited magnetic field systems 523 3.5, Torque preduction in rotating machines 526 3.6. General analysis of electromechanical system 627 Highlights 534 Objective Type Questions 534 Theoretical Questions 535 Unsolved Examples eae 535, MISCELLANEOUS EXAMINATIONS’ QUESTIONS—WITH SOLUTIONS 537—567 LABORATORY EXPERIMENTS 569—619 TO ALMIGHTY 1 D.C. Circuits and Network Analysis 1, Definitions of important terms. 2. Limitations of Ohm's law. 3. Kirchhoff's laws, 4. Applications of Kirchhoff's laws : Branch-current method—Maxwell’s loop (or mesh) current method-——Nodal voltage method. 5, Solving equations by derminants—Cramer’s rule. 6. Superposition theorem. 7. Thevenin’s theorem. 8. Norton's theorem, 9. Maximum power transfer theorem, 10. Delta star transformation, 11. Compensation theorem, 12. Reciprocity theorem. 13, Millan's theorem— Highlights—Objective Type Questions—Theoretical Questions—Unsolved Examples. 1. DEFINITIONS OF IMPORTANT TERMS 1. Circuit. A conducting path through which an electric current either flows or is intended to flaw is called a circuit. The various elements of an electric circuit are called parameters (e.g. resistance, inductance and capacitance). These parameters may be distributed or lumped. 2, Linear circuit. The circuit whose parameters are constant (i.e. thcy do not change with voltage or current) is called a linear circuit. 3. Non-linear circuit. The circuit whose parameters change with voltage or current is called a non-linear circuit. 4, Unilateral circuit. A unilateral circuit is one whose properties or characteristics change with the direction of its operation (e.g. diode rectifier). 5. Bilateral circuit. It is that circuit whose properties or characteristics are same in either direction (e.g. transntission line), 6. Electric network. An electric network arises when a number of parameters or electric elements coexist or combine in any manner or arrangement. 7. Active network. An active network is one which contains one or more than one sources of emf. 8. Passive network. A passive network is one which does not contain any suurce of e.m.f. 9, Node. A node is a junction in a circuit where two or more circuit elements are connected together. 10. Branch. The part of a network which lies between two junctions is called branch. 2, LIMITATIONS OF OHM'S LAW Ina series circuit or in any branch of a simple parallel circuit the calculation of the current is easily effected by the direct application of Ohm’s law. But such a simple calculation is not possible ifono of the branches ofa parallel circuit contains a source of e.m.f., or ifthe current is to be calculated in a part of a network in which sources of e.m.f. may be present in several meshes or loops forming the network. The treatment of such cases is effected by the application of fundamental principles of electric circuits. These principles were correlated by Kirchhoff many years ago and enunciated in the form of two laws, which can be considered as the foundations of circuit analysis. Other, later, methods 1 ¢ 2 ELECTRICAL TECHNOLOGY have been developed, which when applied to special cases considerably shorten the algebra and arithmetic computation compared with the criginal Kirchhoff’s method. 3. KIRCHHOFF’S LAWS For complex circuit computations, the following two laws first stated by Gutsav R. Kirchhoff (1824-87) are indispensable. First law (Point or current law). The sum of the currents entering a junction is equal to the sum of the currents tcaving the junction. Refer Fig. 1. Ifthe currents towards a junction are considered le positive and those away from the same junetion negative, then this law states that the algebraic sum of all cur- rents meeting at a common junction is zero. Is, Ty ie. Currents entering = E currents leaving Ntlgelothy tls cal ter)] or Ay+ig-lp-1y-I5=0 ew [1 (691 Second law (Mesh or voltage law). The sum of the e.m.fs (rises of potential) around any closed loop of I a circuit equals the sum of the potential drops in that loop. Considering a rise of potential as positive (+) and Junction a drop of potential as negative (-}, the algebraic sum of potential differences (voltages) around a closed loop ofa Fig. circuit is zero : EE -XIR drops = 0 (around closed loop) be. LE = LR wl (a)] or = Potential rises = £ potential drops AZ (6) To apply this law in practice, assume an arbitrary current direction for each branch current. The end of the resistor through which the current enters, is then positive, with respect to the other end, Jf the solution for the current being solved turns out negative, then the direction of that current is opposite to the direction assumed. Tn tracing through any single circuit, whether it is by itself or a part ofa network, the following: rules must be applied : 1. A voltage drop exists when tracing through a resistance with or in the same direction as the current, or through a battery or generator against their voltage, that is from positive (+) to negative (-), Refer Fig. 2. Fall in voltage (-E) Fig. 2 2. A voltage rise exists when tracing through a resistance against or in opposite direction to the current or through a battery or a generator with their voltage that is from negative (-} to positive (+), Refer Fig. 3. DC. CIRCUITS AND NETWORK ANALYSIS 3 L - v M rarer vn —— L er Mlustration. Consider a circuit shown in Fig. 4. Considering the loop ABEFA, we get -HR,-15R3+E,=0 or BE, =1R, + isRs (whore Jy = J; +I) ..Ai) Considering the loop BCDEB, we have IpRy - Ey + IyRy = 0 wld) or Eg = IpRp + Igy If Ey, Es, Ry, Re and Ry are known, then Jy, Tg and Jy can be calculated from eqns. (i) and (ii) Fig. 4 4. APPLICATIONS OF KIRCHHOFF'S LAWS. Kirchhoff's laws may be employed in the following methods of solving networks : 1, Branch-current method 2. Maxwell's loop (or mesh) current method 3. Nodal voltage method. 4,1, Branch-Current Method. For a multi-loop circuit the following procedure is adopted for writing equations : 1. Assume currents in different branch of the network. 2. Write down the smallest number of voltage drop loop equations so as to include all circuit elements ; these loop equations are independent. If there are n: nodes of three or more elements in a circuit, then write (n ~ 1) equations as per current law. 3. Solve the above equations simultaneously, The assumption made about the directions of the currents initially is arbitrary. In case the actual direction is opposite to the assumed one, it will be reflected as a negative value for that current in the answer. The branch-current method (the most primitive one) involves more labour and is not used except for very simple circuits. 120 6n Example 1. Jn the circuit of Fig. 5, find the current through each resistor and voltage drop across each resistor. 12 ton 40V Fig. 6 4 ELECTRICAL TECHNOLOGY Solution. Let the currents be as shown in Fig. 6. A 120 B 62 c Fig. 6 Applying Kirchhoff's voltage law to the circuit ABEFA, we get = 12h, - 10Uy + Jy) + 1250 = 22%, - 107, + 12=0 or +11, +5l,-6=0 wei) Cireuit BCDEB gives, 6Fy - 10 + 100, +12) =0 10/, + 16/;-10=0 5, + Bl, -5 =0 ii) Multiplying eqn. (i) by 5 and eqn. (ii) by 11 and subtracting, we get 55/, + 251,-30=0 551, + 88Ip-55=0 - - + - 63/y + 25 =0 ie. Ig =0.397A Substituting this value in eqn. (i), we get 11, +6 0.397-6=0 ie. 1, = 0.365 A Hence, Current through 12 © resistor, 1, = 0.365 A. (Ans.) Current through 6 { resistor, 1, = 0.897 A. (Ans.) Current through 10 9 resistor, I, +1, = 0.762 A. (Ans.) The voltage drop across : 12 Q resistor = 0.965 x 12 =4.38 V. (Ans.) 6 Qresistor = 0.397 x 6 = 2.38 V. (Ans.) 10 Q resistor = 0,762 x 10 = 7.62 V. (Ans.) Example 2. Find the magnitude and direction of currents in each of the batteries shown in Fig. 7. 30V 4a 120: 8a ov 200 tov 20. Fig.7 BC. CIRCUITS AND NETWORK ANALYSIS 5 or or or Solution, Let the directions of currents J), /, and Jy in the batteries be as shown in Fig. 8. Pig. 8 Applying Kirchhoff's voltage law to the circuit ABCFA, we get 30 — 41, —8f, + 20/)~ 20-127, =0 = 24h; + ly +10 =0 12, = 10y~5 = 0 wali) Cirevit ECDEF gives, 20 = 20fy ~ 12U, + fg) + 10=0 20 - 20F)— 12%, - 127, + 10 = 0 - 121, - 32%, + 30 =0 Gly + 16f,- 15 =0 di) Multiplying eqn. (ii) by 2 and subtracting it from (i), we get = 42g +25=0 [y= 0.595 A Substituting this value of Jy in eqn. (i), we get 12), - 10 « 0.595 -5 = 0 40.9124 Hence current through, 30 V battery, 20.9124. (Ans), 20 V battery, J, = 0.595 A. (Ans) 10 ¥ battery, Uy + I) = 1.507 A, (Ans), Example 3. The terminal resistances of batteries A and B are 2.5 Qand 2 Q respectively. The battery has an e.m.f. of 20 volts. A resistance of 10 Q is connected across the battery terminals, Calculate : Le. Le (i) The discharge current of battery A, the discharge current of battery B being 1.75 A. Ui) The emf. of battery B. (iii) The energy dissipated in 10 Q resistance in 46 minutes, Solution. Refer Fig. 9. Wir Applying Kirchhoff's voltage law to the circuit LMNPSQL 20-251, + 1.75x2-E,=0 Eg + 2.51, - 23.5 =0 ol Cireuit LMNPTL gives, . 20 - 2.54, - 100, + ip) = 0 20 2.51, - 100, + L.75)=0 6 ELECTRICAL TECHNOLOGY Fig. 9 or 20 - 12.52, ~ 17. or 1,202A. Hence discharge current of battery A =1, = 0.2 A. (Ans.) ti) Eye Substituting the value of [, in eqn. (i), we get Eg + 2.5% 02-235 =0 7 E,=23V Hence e.m.f. of battery B, Ey = 23 V. (Ans.) Wii) Energy dissipated in 10 9 resistor: Energy dissipated =PRt =U} + fy)" x 10 x (40 x 60) joules = (0.2 + 1,75)? x 10 « (40 x 60) joules = 91260 joules. (Ans.) Example 4.A battery having ane.nf. of 110 Vand an internal resistance of 0.2.Q is connected in parallel with another baitery with emf. of 100 V and a resistance of 0.25 2. The two in parallel ary placed in series with a regulating resistance of 5 ohms and cannected across 220 V mains. Calculate = (i) The magnitude and direction of the current in each battery. (it) The total curreat taken from the mains supply. Solution. Refer Fig. 10. D.C. CIRCUITS AND NETWORK ANALYSIS 7 Wh tylee Let the directions of flow of currents 7; and Jz be as shown in Fig. 10. Applying Kirchhoff's voltage law to LMNPSQL, we get 110 + 0.21; - 0.2615 - 100 = 0 0.2, - 0.251 or 1, - 1.251, =- 50 wali) Cirevit LMNPTL gives, 110 + 0.21, - 220 + BU, +) =0 5.20, +61, = 110 Fy + 0.961, = 21.15 Ail) Subtracting (ii) from (i), we get 2.21 =- 71.15 ” 1,=32.19A. (Ans.) and 1,=-9.75 A. (Ans.) Since /; turns out to be negative, its actual direction of flow is opposite to that shown in Fig. 10, In other words it is not a charging current but a discharging one. However, I is a charging current. (ii) (+1) + The total current taken from the mains supply, A, +1, =-9.75 + 32.19 = 2244 A. (Ans.) Example 5. In the circuit shown in the Fig. 11 determine : (i) All the currents in the network, (ii) Voltages between the points. Solution. Refer Fig. 11. (el) H (yt htiy) 1.562 Fig. 11 (i) All the currents in the network: Let the directions of the currents be as shown in the Fig. 11. Applying Kirchhoff’s voltage law to the circuit BCDHB, we get 0.4fy — 10 — 1.5/3 + 3(, + fp) =0 he, 31, + 3.412 - 1.51; = 10 wali) be, get Le. ELECTRICAL TECHNOLOGY Circuit HDEFH gives, 1.Blg — 6 + O.8(lo + 15) + GU, + Ly +15) = 0 61, + 6.31g + 7.813 = 6 = ii) Circuit ABHFGA gives, — 18, — BU, + Ig) — 6U + Ig 41g) + 12=0 — 10.5; - 9/2 - GIy +12 =0 10.51) + 9g + Gly = 12 Abii) Multiplying eqn. (ii) by 2 and subtracting eqn. (ii) from eqn. (i), we get 0.51; - 10.8/3 = 14 Multiplying eqn. (ii) by 10.6 and eqn. (iii) by 6 and subtracting eqn. (iii) from eqn. (ii), we get 12.18], + 45.9]; =-9 wv) Multiplying eqn. (iv) by 12.15 and eqn. (uv) by 0.5 and subtracting eqn. (v) from eqn. (i - 154,17 13 = 174.6 Ty =-1.132A Substituting the value of Is in eqn. (iv), we get 0.522 - 10.8 (- 1.132) = 14 Ty = 3.549 A Substituting the values of Jz and Jy in eqn. (iii), we get 10.57; + 9 x 3.549 + 6 x (- 1,132) = 12 dy, =-1252A Hence the directions of Jy and fy are actually opposite to the assumed directions. The current between Band H =I, + Ig = — 1.252 + 3.549 = 2.297A. (Ans.) The current between H and F = 1, + fg + Iz =—- 1.252 + 3.549 — 1.132 = 1.165 A. (Ans.) The current between H and F (and E) = J, +J; = 3.549 - 1.132 = 2.417 A. (Ans.) (ii) Voltage between the points: Voltage across BH =2.297x3=6.891V. (Ans.) Voltage across HF = 1.165 x6=6.99 V. (Ans.) Voltage across CE = 12-(- 1.252 x 1.5) = 13.878 V. (Ans.) Example 6, Determine the magnitude and direction of flow of current in the branch MN for the circuit shown in Fig. 12. ev Fig. 12 Solution. Refer Fig. 13. Let the dircetions and magnitudes of the currents flowing in the various circuits be as shown in Fig. 13. D.C. CIRCUITS AND NETWORK ANALYSIS 9 or or or toN) Fig. 13 Applying Kirchhoff's voltage law to the circuit LMNL, we get = 104; = 50/5 + 25/2 =0 1, = 2.519 + 5/3 =0 =) Circuit MPNM gives, — 15UEy — Ig) + 20Ug + 15) + 50Iy = 0 = 15I; + 20/2 + 851, = 0 I, ~ 1.831, - 5.66) = 0 ..fié) Circuit LNPQL gives, ~— 25ly — 20g + 13)+ 8 =0 = 481) - 20/5 +8=0 Ty + 0.4423 = 0.177 es ldié) Subtracting eqn. (ii) from eqn. (i), we get —1.17Iy + 10.6673 = 0 Iy-9.1Uy=0 waliv) Subtracting eqn. (iv) from eqn. (iii), we get 9.55], = 0.177 Jy = 0.0185 A. Hence magnitude of current (I;) flowing through the branch MN = 0.0185 A (from M ). (Ans) Example 7. Determine the current in the 4 Q resistance of the circuit shown in Fig. 14. (Indore University) 20 Fig. 4 10 ELECTRICAL TECHNOLOGY Solution. Refer Fig. 15. al +8) 4 20 M 10M a-l) q-1,-6)P (d= 1-1-6) 40 24V Fig. 15 Let the directions of various currents in different circuits be as shown in the Fig. 15. Applying Kirchhoff's voltage law to the circuit SQLPS, we get = 2, + 10g + 17-1, -6) 20 I= 31, + 10Ig = 6 eal) Circuit LMNPL gives, = 2h +1, + 6)- 10430 -1,- I) - 6) = 10l,=0 87-61, - 15%, = 40 wf) Circuit SPNTS gives, = -1,-6)- 30-1) - 1) -6)-41 + 24=0 ~B81 + 4h +3ly 2-48 or BI - 41; - 3g = 48 .diié) Multiplying eqn. (i) by 3 and subtracting eqn. (ii) from eqn. (é), we get — Al, + 46lg = - 22 or 7) = 11.25fg = 5.5 wwliv) Multiplying eqn. (i) by 8 and subtracting eqn. (iii) from eqn. (i), we get — 20%, + 83%, = 0 wal) Multiplying eqn. (iv) by 20 and adding eqn. (v), we get ~ 142g = 110 o Ig=-O.774A and i=-3.212A Substituting the values of J; and Jz in eqn. (i), we get (-3 x (-3.212)4 10 x(-0.774) = 6 Le. 1= 23.37 A. (Ans.) D.C. CIRCUITS AND NETWORK ANALYSIS MW Example 8. Determine the current in each of the resistors of the network shown in the Fig. 16. Solution, Refer Fig. 16. Let the current directions be as shown in the Fig. 16. Applying Kirchhoff's voltage law to the circuit ABDA, we get ~ 3, - 83 +4I,=0 or BI; - 4Iy + Bly = 0 wali) Circuit BCDB gives, = Bily — Jy) + 2 + 1g) + Bly 20 — 5, + 2p + 15/3 = 0 SI) - 22-16]; = 0 enol) Circuit ADCEA gives, = Aly Uy + Ig) +2=0 — 6ly- 21; =-2 or Bg+lg=1 Addi) Multiplying eqn. (i) by 5 and eqn. (ii) by 3 and subtract- ing (i) from (i), we get 15], - 201; + 401; =0 15]; - GI — 4515 =0 = + _ + - Mg + 85ig = 0 or 142, - 8513 = 0 wali) Multiplying eqn. (iii) by 14 and eqn. (iv) by 3 and subtracting (iv) from (ii), we get 421, + 14[g = 14 42Iy = 25515 =0 =—_+___ 2691, = 14 os Ty = 0.052 A From eqn. (iv), fg =0.316A From eqn. (i), 1, = 0.283 A Hence, Current through 3 Q resistor = I) = 0.289 A. (Ans.) Current through 4 Q resistor =I, = 0.316 A. (Ans.) Current through 8 Q resistor = Iz = 0.052 A. (Ans.) Current through 5 Q resistor = /; -J;= 0.231 A. (Ans,) Current through 2 Q resistor = [2 + J; = 0.368 A. (Ans.) Example 9. Determine the branch currents in the network of Fig. 17. Solution. Refer Fig. 17. Let the current directions be as shown. Applying Kirchhoff's voltage law to the circuit ABBA, we get $-I)x1-Igx14igx1=0 or f-lInp+ty=5 wi) Circuit BCDB gives, = (Uy dg) x 145 + lp 4+ Ig) 1 415% 120 -Kh+h+iy=-5 or h-h-3=5 Ait) Circuit ADCEA gives, —Iy* 1 Uy + ig) 1410-1 +) x 120 12 ELECTRICAL TECHNOLOGY 121,41), Fig. 17 -h-8h-Iy=-10 or dy + 3g +13 = 10 Subtracting (ii) from (i), we get, i,=0 From (i), we have, h-Ip= and — from (iii), we have, 1, +3, = 10 By solving (iv) and (v}, we get, p= 125A and 4, =6.25A Hence, Current in branch AB = 6.25 A. (Ans.) Current in branch BC = 6.25 A. (Ans,) Current in branch BD = 0. (Ans.) Current in branch AD = 1.25 A. (Ans.) Current in branch DC = 1.25 A. (Ans.) Current in branch CA = 7.5 A. (Ans.) Example 10. Find the current in the galuanometer arm of the wheatstone bridge shown in Fig. 18. D.C CIRCUITS AND NETWORK ANALYSIS. 13. Solution. Refer Fig. 18. Let the current directions be as shown. Applying Kirchhoff’s voltage law to the circuit ABDA, we get = 10002, ~— 500/5 + 100/, = 0 1,-0.1, + 0.515 =0 Circuit BCDB gives, — 999061; fs) + 1000619 + Ig) + 50025 = 0 = 99901, + 100%) + 114905 = 0 14 - 0,100.1 - 1.15f5 = 0 aati) Circuit ADCEA gives, = 100/2 = 10001T, + 15) + 20 — 600i, + fy) = 0 - 500!, - 16002, - 1000/3 = - 20 Ty + 8.2ly + 2s = 0.04 Adit} Subtracting (ii) from (i), we get 0.000L> + 1.65%, =0 tive Subtracting (ii) from (ii), we get + 3.300Lg + 8.1515 = + 0.04 wl) Solving (iv) and (v), we get, Jy = 0.735 x 10° A Hence current in the galvanometer arm = 0.735 pA. (Ans.) Example 11. Determine the current supplied by the battery in the circuit shown in Fig. 19. (Bombay University) Solution. Refer Fig. 19. 100.0 L 1002 4 ly ‘T_i200 V (+t) Poteet Fig. 19 Applying Kirchhoff's voltage law to the circuit LMNL, we get — 100; — 30013 + 600/2 = 0 1, -5lg + 3Ig=0 sal) Circuit MNPM gives, = 30025 - 100UIy + 15) + 5001, — 1g) = 0 500/; — 100%; - 900/3 = 0 Ty - 0.21, — 1.81 = 0 wii) Circuit LMPTL gives, ~ L001; = 5001; ~ J) + 200 - 100K; + Ig) = 0 i4 ELECTRICAL TECHNOLOGY = 1001, - 100, + 50023 = ~ 200 1, + 0.143] - 0.7141, = 0.286 wld) Multiplying (i) by 1.8 and (ii) by 3 and adding, we get 1.81; - 91, + 5.415 = 0 81; ~ 0.61g~ 5.4ly = 0 4.81; — 9.61 =0 or 1, -2fg=0 wwliv) Multiplying (2) by 0.714 and (iii) by 1.8 and subtracting, we get 0.71417, - 0,143%y - 1.28515 = 0 BI; + 0.25719 — 1.286/3 = - - + - = 1.086/; — 0.4lg or Jy + 0.368fp = 0.474 elo) Subtracting (o} from (iv), we get -2.368/y = 0.474 ie. Ip=0.2A and N=04A « Current supplied by the battery = J, +/.=0.2+046=06A. (Ans) 4.2. Maxwell's Loop (or Mesh) Current Method The method of Joop or mesh currents is generally used in solving networks having some degree of complexity. Such a degree of complexity already begins for a network of three meshes. It might even be convenient at times to use the method of loop or mesh currents for solving a two-mesh circuit. The mesh-current method is preferred to the general or branch-current method because the unknowns in the initial stage of solving a network are equal to the number of meshes, .e., the mesh currents. The necessity of writing the node-current equations, as done in the general or branch- current method where branch currents are used, is obviated. There are as many mesh-voltage equations as these are independent loop or mesh, currents. Hence, the M-mesh currents are obtained by solving the M-mesh voltages or loop equations for M unknowns. After solving for the mesh currents, only a matter of resolving the confluent mesh currents into the respective branch currents by very simple algebraic manipulations is required. This method eliminates a great deal of tedious work involved in branch-current method and is best suited when energy sources are voltage sources rather than current sources, This method can be used only for planar circuits, The procedure for writing the equations is as follows : 1, Assume the smallest number of mesh currents so that at least one mesh current links every element. As a matter of convenience, all mesh currents are assumed to have a clockwise direction. The number of mesh currents is equal to the number of meshes in the circuit. 2. For each mesh write down the Kirchhofi’s voltage law equation. Where more than one mesh current flows through an element, the algebraic sum of currents should be used. The algebraic sum of mesh currents may be sum or the difference of the currents flowing through the element. depending on the direction of mesh currents. 3, Solve the above equations and from the mesh currents find the branch currents. Fig. 20 shows two batteries £, and E, connected in a network consisting of three resistors. Let the loop currents for two meshes be J, and J, (both clockwise-assumed). [t is obvious that current through 3 (when considered as a part of first loop) is (/, — fy), However, when Rg is considered part of the second loop, current through it is Uy ~J,). D.C. CIRCUITS AND NETWORK ANALYSIS 15 Applying Kirchhoff's voltage law to the two loops, we get Ey -1)R,-Ryhy- 12) = 0 or Ey - TR, + Ry) + 1gRy = 0 ww loop 1 Similarly, ~IgRy - Eg - Rollg-1y) =0 =1,Rz- Ez = 1oRy + )R3=0 or 1,Rz— Ih Ry + R3)- Ey = 0 sdoop 2 Fig. 20 The above two equations can be solved not anly to find loop currents but branch currents as well. Example 12. Determine the currents through various resistors of the circuit shown in Fig. 21 using the concept of mesh currents. Solution. Refer Fig. 21. Since there are two meshes, let the loop currents be as shown, Applying Kirchhoff's law to loop 1, we get 24 — 41; - 2, — I) = 0 ~ 6, + 21, +24=0 avid 20 hy 12V Bh-k=12 ..{i) For loop 2, we have = 2Uly- 1h) -6Iy- 12=0 25, -8lg-12=0 P -4h=6 Mi) L 49 M 6a or Fig. 21 Solving @ « 42 ving (i) and (i), we get, y= TTA 6 and n= -7y4 Hence Current through 4.9 resistor = 42 A (from Lito M). (Ans) Current through 6 0 resistor = © A (from N to M). (Ans.) Ourren through 20 resistor = 7 -[ ~&}tr4 (from M to P). (Ans.) 16 ELECTRICAL TECHNOLOGY Example 13, Determine the current supplied by each battery in the circuit shown in Fig. 22. (Aligarh University) $a 40 ,5y 6 Solution. Refer Fig. 22. As there are three meshes, let the three loop currents be as shown. Applying Kirchhoff's law to loop 1, we get 20-5) - Hy -)-5=0 or 87, - 3%, = 15 valid For loop 2, we have = 40y + 5 = 20g - 1g) +5 45-3 —-dy BI, ~ 9[q + 2g = ub For loop 3, we have = Bl - 30 - 5 - 2g —1y) = 0 2p ~ 10g = 85 Ati) Eliminating /; from (i) and (ii), we get 63fy - 16/y = 165 vali) Solving (iii) and (iv),we get Ip= 1.82 A and Ig =-3.15 A (ve sign means direction of current is counter-clockwise) Substituting the value of Jz in (i), we get N=256A Current through battery B, (discharging current) = J, = 2.56 A. (Ans.) Current through battery By (charging current) = J, - Jy = 2.56 - 1.82 = 0.74 A. (Ans.) Current through battery By (discharging current) = Ip + Jy = 1.82 +3.15 =4.97A. (Ans.) Current through battery B, (discharging current) = J, = 1.82 A. (Ans.) Current through battery B; (discharging current) = J, = 3.15 A. (Ans.) Example 14. Determine the currents through the different branches of the bridge circuit shown in Fig. 23. Solution. Refer Fig. 23. The three mesh currents are assumed as shown, D.C. CIRCUITS AND NETWORK ANALYSIS or or or or (Ans.) The equations for the three meshes are : For loop I: 240 - 2017, - 1) - 607, -I3) = 0 - 70M, + 20%5 + 50%, = - 240 TOL, — 20%, - 50fg= 240 Ki) For loop 2: - 30% — 40(I2 — 13) - 20, - 1) =0 20%, — 901, + 401, = 0 2, -91g+4Ig20 ii) For loop 3; — 803 — 50(Tg ~I,) - 4023 — Iz) = 0 5O!, + 40/)-150%;=0 . Bi, +4Ig-15!,=0 Midi) Solving these equations, we get 1, =6.10 A, Jp = 2.56 A, Jy = 2.724 Current through 30 Q resistor = f, = 2.66 A (A to B). Fig. 23 Current through 60 Q resistor = J, = 2.72 A(BtoC), (Ans.) Current through 20 Q resistor = J, — J, = 6.10- 2.56 =3.54 A(Ato D). (Ans.) Current through 50 Q resistor = J, — J; = 6.10 - 2.72 = 3.38 A(Dto C). (Ans.) Current through 40 Q resistor = I; -— Jz = 2.72 — 2.56 = 0.16 A(D to B). (Ans.) 4.3, Nodal Voltage Method Under this method the following procedure is adopted : 1. Assume the voltages of the different independent nodes. 2. Write the equations for each mode as per Kirchhoff's current law. 3. Solve the above equations to get the node voltages. 4, Calculate the branch currents from the values of node voltages. . Let us consider the circuit shown in the Fig. 24 L and M are the two independent nodes ; M can be taken as the reference node. Let the voltage of node Z (with respect to M) be V,. Using Kirchhoff's law, we get }#-—— V, ——+e—— v, ——+} hehzh (8) t Ohm's lawgives I= z = eM he Vo _(B2- Vi) Ry Ry Ip M82 ME E,-Vi | Eo- Vi _ Vi Ry Ry Ry Rearranging the terms, we get A,2,2) 1% vl atm tm Ren? 16) AB) 18 ELECTRICAL TECHNOLOGY It may be noted that the above nodal equation contains the following terms : (i) The node voltage multiplied by the sum of all conductances connected to that anode. This term is positive. (ii) The node voltage at the other end of each branch (connected to this node) multiplied by the conductance of branch. These terms are negative. — Inthis method of solving a network the number of equations required far the solution is one less than the number of independent nodes in the network. — Ingeneral the nodal analysis yields similar solutions. — The nodal method is very suitable for computer work. Example 15. For the circuit shown in Fig. 25 find the currents through the resistances Ry and Ry. Ry L Re um. Bs —,— sov 109 E, = 20V¥ Fig. 25 Solution. Refer Fig. 25. Let L, M and N = independent nodes, and V, and Vy = voltages of nodes L and M with respect to node N. The nodal equations for the nodes L and M are : A,t 4) 21 Ve vl at a | 5 Ry oli) 2,1,1)_2_¥ i Vul atte i [RE voll) Substituting the values in (i) and (ii) and simplifying, we get Lil, 1) 50 Vu vi (S+a*an}- - =0 or 2.5V, - Vig - 50 =0 Adi) and vu gtintd|-3-32° or 2.5Vy - Vi, - 2020 or -V, + 2.5Vy-20=0 efi) Solving (ii) and (iv), we get Vi = 27.6 V, Vi = 19.05 V Vz _ 27.6 Current through Rg = R 07 2.76A. (Ans.) Current through Ry = Yu = Me 4 = 1,906 A. (Ans.) ‘D.C. CIRCUITS AND NETWORK ANALYSIS 19 5. SOLVING EQUATIONS BY DETERMINANTS—CRAMER'S RULE If the number of equations is more than two, it is easier to get the solution by using determinants. 1€7,, fg and /y are the three unknowns jn a system of three linear equations Gayl + Gyply + Gygly = Cy agit) + azaly + angly = Cy Gaul + s2l2 + azals = Cy an rane ae ee ee ote = Ste ° the values fy, fy and fare given by “1 G2 Sy 2 Gm az 3 83 «8x3 4, val (a) 1 1 a2, 2 ag 3 fgg 2. a [9 )) ay ay ey a2 S22 Oe am (932 a Is= — el Ce)] a1 6912 (O13 where Asj}a7 a7) a273 a3: «93233 This is known as Cramer's rule and can be applied to any system ofn linear equations provided Ais not zero. This theorem is sometimes useful in solution of networks in which some branches may contain sources of e.m.f. It is applicable only to linear networks where current is linearly related to voltage as per Ofvm's law. This theorem may be stated as follows : “In any network containing more than one source of e.m.f. the current in any branch is the algebraic sum of a number of individual fictitious currents (the number being equal to the number of sources of e.m.f.), each of which is due to the separate action of each source of e.m.f,, taken in order, when the remaining sources of e.m.f. are replaced by conductors, the resistances of which are equal to the internal resistances of the respective sources”. ‘The procedure of applying superposition theorem is as follows : 1, Replace al! but one of the sources by their internal resistances. If the internal resistance of Hae ccaie. Szcompered to other resistances present in the network, the source is replaced a cireuit, 20 ELECTRICAL TECHNOLOGY 2. Find the currents in different branches by using Ohm's law. 3. Repeat the process using cach of the e.m.fs. as the sole e.m.f. each time.” ‘The total current in any branch of the cireuit is the algebraic sum of currents due to cach source, When finding total current in any branch, it is necessary te take into account the directions of the currents caused by each individual source, currents flowing in the same direction being additive, currents flowing in opposite directions being subtractive. Explanation: — InFig. 26, 1;, fz and J represent the values of currents which are due to the simultaneous action of the two sources of emf. in the network. — Inthe Fig. 27 are shown the current values which would have been obtained if left-hand side battery had acted alone. — Similarly Fig. 28 represents conditions obtained when right-hand side battery acts alone. By combining the current values of Fig. 27 and 28, the actual values of Fig. 26 can be obtained > — _", —", Ry Py . 3] *, * Fig. 27 Obviously, hely-i" Ty = Ig" = by’ [=I +I", Example 16, By using superposition theorem find the currents in the different branches of the network shown in Fig, 29. \— —|, M 2 E,=20V 2vV=—e Fig. 29 Solution, I, :, Ig First step. Refer Fig. 30. Take e.m.f. £; only and replace e.m.f, £9 by its zero internal resistance, the circuit is shown in Fig. 30. D.C CIRCUITS AND NETWORK ANALYSIS 21 wire om wl ow 6Q 102 Ej=zov 120 r E,=28V Q Q Fig. 30 Fig. 31 ‘Total resistance =B+ iz * “ 213.452 Current through 8 Q resistance, Tyo Boye 1as7 a Current through 10.9 resistanct, | 12 y= 1487 «0 qo 781A Current through 6 Q resistance, . 10 P= 148IxT 19 70-8754 Second step. Refer Fig. 31. E.m.f. E, is removed/short circuited and current due to e.m.f. £2 is found. The current is shown in the Fig. 31. Total resistance =10+ Current through 10 Q resistance, » 28 By = jpg 1802 A Current through 8 Q resistance, 12 Ty = 1.892% 757g = L135 A Current through 12Q resistance, I" = 1.892% 55g = OETA ‘The total currents in different branches are : Current through 8 Q resistance, Ty = 1 - 2)" = 1.487 - 1.135 = 0.352 A (from L to M). (Ans.) Current through 10 2 resistance, 1, = |,” = Iy' = 1.892 - 0.81 = 1.082 A (from N to M). (Ans.) Current through 12 .Q resistance, l=f' + J" = 0.676 + 0,757 = 1.432 A (from M to Q), (Ans.) 2 ELECTRICAL TECHNOLOGY Example 17. By using superposition theorem, find the current in 2 Q resistance shown in Fig. 32. Internal resistances of the cells are negligible. ord yy #1¥ Pig 32 Fig. 33 Solution. First step. Refer Fig. 33. — E,has been removed. — Resistances 2 0 and 0.08 Q are in parallel across points L and N. 20.08 R= 9,008 ‘This resistance is in series with 0.1-0. Hence, tota] resistance offered ta battery E; =0076 2 #0.1+0,076 = 0.1762 ». Current, 1a Ad eBasa Current through 2 Q resistance, 0.08 h=33* tos, 3 70.896 A (from N toL) Second step. Refer Fig. 34. — £, has been removed. — Combined resistance of paths NML and NQL = 2201 0.095 0 Total resistance offered toZ, = 0.095 + 0.08 = 0.175 0 . Current 1a AS 0 2457A Again, hhems7xs2t gt llTA Hence total current through the 2 9 resistance when both batteries are present = J; + J_= 0.896 + 1.17 = 2.066.A. (Ans.) D.C. CIRCUITS AND NETWORK ANALYSIS 23 Example 18. Using superposition theorem, find the currents in the circuit shown in the Fig. 35. 4 ov sa | E, . Fig. 35 Solution. First step. Refer Fig. 36. — &; has been removed. — Resistances Ry and Rs are in parallel, Fig. 36 This resistance is in series with Ry. + Total resistance offered to battery Ey =857+5213572 Current hy = 5gg7 72214 F —b_ Current dy =-2.21 * 30415 0.947 A Current Fy = 2.21 Xoo0457 1.263 A. Second step. Refer Fig. 37. — £E, has been removed. — Resistances R; and Ry are in parallel, . RiRy _ 5x15 ” Roaral = Fa Fe erg 78752 ‘This resistance is in series with 20 Q resistance. 24 ELECTRICAL TECHNOLOGY . Total resistance offered to Ey 23. 75 +202 23.75. 0 Current i= x oe =5.05A Current fy = 5.05 x 78 = 3.79.8 Current Ig = 6.05 x 5 s = 1.2624 ‘Now superimposing the results, we get dyet +0)" = 2.21+(-3.79 A)=- 158A. (Ans.) y= Iq! + Ty” = - 0,947 + 5.05 = 4,1 A. (Ans) dy = Ty + 1s” = 1.263 + 1.262 = 2.52 A. (Ans.) 7. THEVENIN'S THEOREM Thevenin's theorem is quite useful when the current in one branch of a network is to be determined or when the current in an added branch is to be calculated. “It states that for the purpose of determining the current.ina resistor, Ry, connected across two terminals of a network which contains sources of e.m.f. and resistors, the network can be replaced by asingle source ofe.m.f. and q series resistor, Ry. Thise.m.f, Ey, ts equal to potential difference between the terminals of the network when the resistor, R, is removed : the resistance of series resistor, R,p, is equal to the equivalent resistance of the network with the resistor, R, remaved (or as it is sometimes called, “the resistance of the network when viewed from the terminals under consideration’). E Hence Tz Reka (10) Explanation. Let us consider the circuit shown in Fig. 38 (a). The following steps are required to find current through the load resistance R,. 1. Remove R, from the cireuit terminals A and B and redraw the circuit as shown in Fig. 38 tb). Obviously the terminals have been open circuited. A, oc Ry a Ry Ry Ee R, RL FR, Ew D B (a) () A, Ry Rn A Fn R, <=> En A (a (4 8 Fig. 38 DC. CIRCUITS AND NETWORK ANALYSIS 25 2. Calculate the open-circuit voltage (V,, = Z,,) which appears across tetminals A and B, when they are open i.e. when R, is removed. This voltage is Z,,, (Thevenin's voltage). A little thought will reveal that ER, Eu= Rah (1) . pF ER? [- T= gE gmt Ea= lee Gre: | 3. Short circuit the battery and find the Thevenin resistance Ry, of the network as seen from the terminals A and & [Fig. 38 (c)] Ryx Ry Ra= Bret Rs wn 12) 4. Connect R;, back across the terminals A and B from where it was temporarily removed earlier [Fig. 38 (d)]. Current through R, is given by Eun as RD (13) Example 19. With reference to the network shown in Fig. 39, by using Thevenin’s theorem find the following : (i) The equivalent e.m.f. of the network when viewed from terminals L and M. (ii) The equivalent resistance of the network when looked into from terminals Land M. (iii) Current in the load resistance Ry, of 30 Q. 6a L 48 30.0(R) M Fig. 39 Solution. (() Equivalent e.m.f. of the network : Refer Fig. 39. Current in the network before load resistance (Ry) is connected 48. =gae679 715A «. Voltage across terminals LM, Vog = Ey, = 24% 1.5 = 96 V. Hence, so far as terminals Z and M are connected, the network has an e.m.f. of 36 V (and not 48 V). (Ans) 60 L 62 L L 6Q 48V $240 Ves Eq te bs =2A :, Voltage drop over 10 Q resistance =2x10=20V Hence Voce = Ei, = 20V ‘The resistance of the circuit as looked into the network from points L and M (when battery has been removed), 2 sera The wholecireuit up toLM can now be replaced by a single source of ¢.m.f. and single resistance as shawn in Fig. 60. BR, = Ry =30+10 | 20= 30+ 32 ELECTRICAL TECHNOLOGY En 20 (i) When Ry = 5, T= er Ry” 3067 +5 =048A, (Ans.) (tt) When Ry, = 25 Q, = 0,324 A, (Ans.) in 0 36.67 +25 According to the maximum power transfer theorem, power drawn by Ry would be maximum when Ry = R, or when Ry = 36.67 2 Maximum power drawn by R, 2 E, E (arte) 8 Eh Ge RusRp) 207 * [x 9667 7 272 W. (Ans.) 10, DELTA STAR TRANSFORMATION When networks having a large number of branches are to be solved by the use of Kirchhoff's law, a great difficulty is experienced in solving several simultaneous equations. Such complicated networks, however, can be simplified by successively replacing delta meshes by equivalent star systems and vice versa, Ray Ry Fig. G2 Consider the two circuits shown in the Figs. 61 and 62. They will be equivalent ifthe resistance measured between any two of the terminals 1, 2 and 3 is the same in the two cases. (Rich = Rrels = 16) _ Rigttes + Ray) or Rit Ras a Rot Ro tT) Rg +R Similarly Ro +Ry= gee 18) Ra(Ri2 + Roa) and Ra + Ri = Re Ra ea 19) Solving eqns. (17), (18) and (19) simultaneously, we get Rik Fat Ras FR “mm RoRia Fe" Fag Rag +R “ay B.C. CIRCUITS ANID NETWORK ANALYSIS 33 ya Paras 3 Rig + Rog + Ray From above it may be noted that resistance of each arm of the star is given by the product of the resistances of the two delta sides that meet at its end clivided by the sum of the three delta resistunces. From eqns. (17) to (19), eqns. for star to delta conversion can also be obtained. These are as follows 22) RyRy + RoRy + Rak Rye 223) Rag = Wie + Paka + Ray (24) Ry RyRy + RyRy + B Hay = MR (98) In electronics, star and delta circuits are generally referred to as t and 7 circuits respectively. Example 26. Fig. 63 shows a number of resistances connected in delta and star. Using staridelta conversion method complete the network resistance measured between (i) L and M (ii) M and N and (iii) N and L. t M 20 N Fig. 63 Solution. Three resistances 12 ©, 6 2 and 8 Q are starconnected, Transform them into delta. with ends at the same points as before. 1 1 Ry, 3120 Ry, Fis aa Ry A, sn 3 a Ry 2 a) a) Fig. G4 Refer Fig. 64. Rik + Ray + Roki Rye = 22*6+6x8+8x12 _ 8 27a 34 ELECTRICAL TECHNOLOGY RyRy + ReRy + RyRy Ry = eee Ry ~12%6+6%8+8% 12 _ bd i =1869 Similarly Ry = ARStOxS sr Ox? =362. L 120 270 M 1.82 N Fig. 65 Pig. 66 Fig. 65 shows this transformed circuit connected to original delta connected resistances in the circuit 18,3 Q and 2Q. Here 18 Q and 36 Q are in parallel ; 3.Q and 27 @ are in parallel, and 2.Q and 18 2 are in parallel. These resistances arc equivalent to: 18x 36 3x27 2x18 | e+ 367 122; 307 7 27 Gand 3718" 182 This is shown in Fig. 66. (i) Resistance between L and M, _ 12% (2.7418) _ Rin= Tp a7e1ay 7 2272 (Ans) (ii) Resistance between M and N, 18 x (12+ 2.7) = Run = Ts, 19427 7162 (Ans) (ii) Resistance between N and L, _ 2.7% (12 +18) _ = O74 (2418) * 2.259. (Ans.) Example 27. In the circuit shown in Fig. 67, find the resistance between M and N. Ry Fig. 67 D.C. CIRCUITS AND NETWORK ANALYSIS 35 Solution. Connecting the 1 23 delta [Fig. 68 (i)] to equivalent star [Fig. 68 (1i)] __ Rls 5x8 Ri> Ravi 542+ 7152 RasRi2 2x5 Ra= Roe Rn +Ry 5+2+5 71% Ra Rog ax2 and Ra= Boy Rg +R b+2+5 082 a @ Fig. 68 Thus the original cireuit reduces to that shown in Fig. 69 which further reduces to the circuit shown in Fig. 70. L L 300 s00 “ 300 nN M ‘30.0 N Fig. 69 Fig. 70 Now, the inner star circuit of Fig. 69 shown as Fig. 71 (i) is equivalent to the delta circuit. shown in Fig. 71 (ii) as appears from calculations given below : = MiRo+ RoRg+ RoR _ 8x10 +10x10+10xB _ 99 Ra Rs 10 Fog = EiRas Patay MaRh _ 8x10 + 10410410028 J 595.0 1 RiRa + RaRy + RoR _ 8x10 +10x10+10%8 _ og 36 ELECTRICAL TECHNOLOGY ui) i) Fig. 71 :. The given circuit thus ultimately reduces to the circuit shown in Fig. 72 which in turn is equivalent to the circuit given in Fig. 73. L 302 OF a 13.90 13.0 2 a Le | N 30.2) M 15.62 N Fig. 72 Fig. 73 In Fig. 72, the observe that : 30 @ and 26 Q are in parallel and are equivalent to: 30 x26 30 +26 30 and 26 Q are in parallel and are equivalent to; 13.9 Q (as above) 32.5 Q and 30 Q are in parallel and are equivalent to: 32.5 x 30 32.5 + 30 Hence total resistance between M and N, Reagy 2 15:8 (18.9 + 15.9) MN 15.6 + (18.9 + 13.9) _ 433.68 ~ 43.4 Example 28, Find the current I supplied by the battery for Fig. 74, using delta/star transfor- mation. = 13.92. =156Q =9.99.2. (Ans.) B.C. CIRCUITS AND NETWORK ANALYSIS 37 209. L 109 15 252 Fig. 74 Solution. Delta connected resistances 25 Q, 10 Q and 15 Q are transformed to equivalent star as given below : (See Fig. 75) L 250 102 Poy Pa Fis M 152 N ab la) Fig. 75 = Ryka = 10x25 =52 Ry + Rag +R3, 104+ 15+ 25 RasRia 15 x 10 Ro" Ras Rog +R 10415 +95 92 = — PaiRas 25 «15 Ryg+Rg+ Ry 10+ 15 +95 77? The given circuit thus reduces to the circuit shown in Fig. 76. P u N 10.29 2 30 Vousy 259 Fig. 76 Fig. 77 38 ELECTRICAL TECHNOLOGY The equivalent resistance of 25 x 17.5 (20 + 5) QI) (10+ 7.5) O = 55-7 = 10.20.02 Thus the given circuit ultimately reduces to the circuit shown in Fig. 77. Total resistance = 10.29 +3 + 2.6 = 15,792 Hence current through the battery, 15 = 75.79 7085 A. (Ans.) Example 29, Find the current in the galuanometer arm in the network shown in Fig. 78, using deltalstar transformation. L 600 200 P @) M aon 400 N av Fig. 78 Solution. Delta connected resistances 100 Q, 20 2 and 40 Q are transformed to equivalent star as given below : (Seo Fig. 79) Rik 20 x 100 Ri= Rye Rog + Rai” 20+ 40+100 7 1252 Rae Rg rie Ry" Deter e8 Rae pri Rew ota 7292 L Lt 100 2, Rn aun Ray N “on M Ww inp Fig. 79 D.C. CIRCUITS AND NETWORK ANALYSIS 39 The given circuit thus reduces to the circuit shown in Fig. 80. av Fig. 80 ‘The equivalent resistance, (60 + 12.5) x (80 + 25) =4299 (60 + 12.5) + (80 + 25) Rey = + Total resistance of the circuit, Roy =42.9+5= 4792 . 4 + The main current, = Fg 7 0.0835 A Let J, = current in the arm NLP, I; = current in the arm LP, and 1, = current in the galvanometer. (80 + 25) (80 + 25) + (60 + 12.5) _ (60 +125) . and y= 0.0835 Oras) ao, Tae = AORTA. Then, Ty = 0.0835 { |- 0.0494 A ". Potential difference between L and P = 0.0494 x 60 = 2.964 V and between P and N = 0.0341 x 80 = 2.728 V and between L and N = 2.964 - 2.728 = 0.236 V Hence the current flowing through the galvanometer, In= oe = 0.00236 A. (Ans.) 11, COMPENSATION THEOREM The compensation theorem is particularly useful for the following purposes : @) To calculate the sensitivity of a bridge network. (ii) To analyse those networks where the values of the branch elements are varied and for studying the effect of tolerance on such values. 40 ELECTRICAL TECHNOLOGY This theorem is stated as follows : “If a change, say OR, is made in the resistance of any branch of a network when the current was originally I, then the change of current at any other point in the network may be calculated by assuming that an em.f. - 1 AR has been introduced into the changed branch while all other sources have their e.m.fs. suppressed and are represented by their internal resistances only". 12. RECIPROCITY THEOREM The theorem is stated as follows : “In any linear bilateral network, if a source of e.m.f. E in any branch produces a current J in any other branch, then the same e.m.f. acting in the second branch would produce the same current I in the first branch”, In other words, it simply means that E and J are mutually transferrable. The ratio £/7 is known as the ¢ransfer resistance (or impedance in a.c. systems). 13, MILLMAN’S THEOREM The theorem can be applied to a network having a combination of voltage and current sources since voltage source can be converted into a current source and vice-versa. The theorem is applicable only to two sources connected directly in parallel. Itis not applicable where there are resistance elements between the sources. The theorem is stated in the following manner : “Any number of current sources in parallel may be replaced by a single current source whose current is the algebraic sum of individual souree currents and source resistance is the parallel combination of individual source resistances”. 1. Kirehhoff's laws: First law : ¥ currents entoring = E eurrents leaving Second law : potential rises = ¥ potential drops. 2, Applications of Kirchhoff's laws include the following : (@) Branch-current method (té) Maxwell's loop (or mesh) current method Gié) Nodal voltage method. 3. The solutions of the networks involve the use of the following theorems : — Superposition theorem — Thevenin's theorem — Nortan’s theorem — Maximum power transfer theorem — Delta star transformation — Compensation theorem — Reciprocity theorem — Millman’s theorem. OC. CIRCUITS AND NETWORK ANALYSIS 41 lL 4 1. 10. Ll OBJECTIVE TYPE QUESTIONS. ‘Choose the Correct Answer Kirchhoff’s current law states that (a) net current flow at the junction is positive (6) algebraic sum of the currents meeting at the junction is zero (ec) no current can leave the junction without some current entering it (d) total sum of currents meeting at the junction is zero. According to Kirebhofl's voltage law, the algebraic sum of all /R drops and e.m4s. in any closed loop of a network is always (a) negative (0) positive (c) determined by battery e.m.fs. (d) vero. Kirchhoff’s current law is applicable to only (a) junction in a network (b) closed loops in a network (e)electric circuits (d) electronic circuits. Kirchhoff voltage law is related to (a) junction voltages (5) battery e.m.fs. (ec) IR drops id) both (a) and (b} (e) none of the above. Superposition theorem can bo applied only ta circuits having (a) resistive elements (b) passive elements (c) non-linear clements (d) linear bilateral elements. ‘The concept on which superposition theorem is based is (a) reciprocity {b) duality (ec) non-linearity (d) linearity. ‘Thevenin resistance Ry, is found (a) by removing voltage sources along with their internal resistances (b) by short-cireuiting the given two terminals (c) between any twa ‘open’ terminals (d) between same open terminals as for E,,. An ideal voltage source should have (a) large value of e.m.f (6) small value of eam.f. (e} zero source resistance (d} infinite source resistance. For a voltage source (a) terminal voltage is always lower than source e.m.f. (b) terminal voltage cannot be higher than source e.in-f, (c) the source e.in.£. and terminal voltage are equal. To determine the polarity of the voltage drop across a resister, it is necessary to know (a) value of current through the resistor (b) direction of current through the resistor (e) value of resistor (d)e.m-fs. in the circuit. ANSWERS () 2. (d) 3. la) 4 (a) G.td) 6 id) Tid) te) 9. tb) 10. (b). 42 8. 10. 1. 12. ELECTRICAL TECHNOLOGY THEORETICAL QUESTIONS Define the following terms : Circuit, Electrical network, Active network, Node and Branch. What are the limitations of Ohm's law ? State and explain Kirchhoff's laws. Discuss briefly application of Kirchhoff's laws. Explain the nodal voltage method for solving networks. How are the nodal equations written ? Explain Cramer's rule used for solving equations by determinants. State and explain superposition theorem. State Norton's theorem. List the steps for finding the current in a branch of a network with the help of this theorem. State Thevenin’s theorem, State the maximum power transfer theorem and explain its importance. State the compensation theorem and discuss its application. State Millman’s theorem. UNSOLVED EXAMPLES Determine the magnitude and direction of the current in each of the batteries ZL, M and N shown in the Fig. 81. 40 6Q 20v sov=* 10 6a Fig. 81 ‘Two batteries are connected in parallel. The e.m.f, and internal resistance of one are 110 V and 6 2 respectively and the corresponding values for other ure 130 V and 4 Q respectively. A resistance of 20.0 is connected across the parallel combination. Calculate : (i) The value and direction of the current in each battery. (ii) The terminal voltage. (Ans, (i) 0.1786 A, 5.2678 A (ii) 108.928 V) (Hint. Terminal voltage = (1, + ,)F) Two cells A and B are connected in parallel, unlike poles being joined together. The terminals of the cells are then joined by two resistors of 4 Q and 2 {2 in parallel. The e.m-f. of A is 2 V, its internal resistance is 19; the emf of B is 1 V, its internal resistance is 2.Q. Find the current in each of the four branches of the circuit, (Ans. 4/3 A, 5/6 A, UG A, 1/3 AJ D.C. CIRCUITS AND NETWORK ANALYSIS 43 4, What is the equivalent resistance of the network shown in Fig. 82? [Ans. 600 O] (B-T.E. Delhi) 6002 100 2 100.2 800.2. 200 0 2002, 80002 100 2 100.2 Fig. 62 &. Determine the currents in the three batteries (2, M and N) in the network shown in Fig. 83 and show their values and direction of flow on the diagram. Neglect the internal resistance of batteries. [Ans 24,14, 0A) av Fig. 84 6. Determine the magnitude and direction of flow of current in the branch CD for the circuit shown in the above Fig. 84. [Ans. 7/755 A from C to D] 7. For the lattice-type network shown in Fig. 85, cal- culate the current. in each branch of the network. 500 a 1000 5 502 (Bangladesh University) [ Amst = 56 43 oes 05% he = 795 son Ly. = e laa Gag Atlas As ‘a i054 | tov ' 500 2 son © ton 4 800 Fig. BS. 4 ELECTRICAL TECHNOLOGY 8, Two storage batteries, A and 8, are connected in parallel to supply a load the resistance of which is 1.2.9, Caloulate : (i) The current in this load. (ii) The current supplied by each battery if the open-circuit e.m.f. of A is 12.5 V and that of B is 12.8 V, the internal resistance of A being 0.05 Q and that of B 0.08 Q. (London University) VAns, (i) 10.25 A (ii) 4 A (A), 6.25 A (BD) 9. A load having a resistance of 0.1 2 is fed by two storage batteries connected in parallel. The open circuit e.m.f, of one battery (A) is 12.1 V and that of the other battery (2) is 11.8 V. The internal resistances are 0.03 2 and 0.04 Q respectively. Calculate: ) The current supplied to the load (ii) The current in each battery (ii) The terminal voltage of cach battery. {London University} (Ans. (7) 102.2 A, (i) 62.7 A (A), 39.5 A (B), (iii) 10.22 V) 10. Abattery havingan e.m.f. of 110 V and an internal resistance of 0.2.0 isconnected in parallel withanother battery having an e.m.f of 100 V and internal resistance 0.25 @. The two batteries in parallel are placed in series with a regulating resistance of 5 0 and connected across 200 V mains. Calculate : (i) The magnitude and direction of the current in each battery. (Gi) The total current taken from the supply mains. (Sumbhal University) {Ans. Gi) 11.96 A (discharge) ; 30.43 A (charge) (ii) 18.47 AJ 11. A wheatstone bridge ABCD is arranged as follows : AB = 50 ohms, BC = 100 ohms and CD = 101 ohms. Agalvanometer of 1000 ohms resistance is connected between 8 and J. A 2-volt battery having negligible resistance is connceted across A and C. Estimate the current flowing through the galvanometer. [Ans. 4.14 A from D to B] 12. Determine the current through the galvanometer G in the wheatstone bridge network of the given Fig. 86, [Ans, 0.52 mA} 500 2 (G)s000 >c L209 w 30 A 200 0 12V 12 6v Fig. 86 Fig. 87 13. Determine the currents through various resistors of the circuit shown in above Fig. 87. ae i 48 [Ame {TAU-toan sty Awvioan: TAP) | DC. CIRCUITS AND NETWORK ANALYSIS 45 14, Find J), J, and J; in the network shown in Fig. 85, using loop-current method. lAns, 14,2 A,3 Al 10.2 202 100 182 10V 21V Fig. 88 Fig. 89. 15. Find the enrrents in the branches of network of Fig. 89 using nodal voltage method. [Ans /, = 1.42 AI, = 1.68 A.J, = 0.96 AJ, = LEAT; = 1.96 A, 1, = 0.32 Al 16, Using superposition theorem find the currents in the different branches of the network shown in Fig. 90, Ams. J, = 0.352 A, f, = 1.082 A, J = 1.432 Aj 21.5V 0.040 . Fig. 91 47. By using superposition theorem, find the current in 19 resistance in Fig. 91, Internal resistances of the cells are negligible. [Ans, 2.066 A] 18. Using superposition theorem find the current in 20 2 resistor of the circuit shown in Fig. 92. JAns, 0.443 Al 302 600 48V 480 722 48 a 60vV—— 72V Fig. 92 Fig. 93 19, Find the current in the 30 Q resistor of the circuit shown in Fig. 93, using superposition theorem. lAns. 0.662 Al a6 ELECTRICAL TECHNOLOGY 20. With reference to the network of Fig, 94, by applying Thevenin’s theorem find the following : (i) The equivalent c.m.f. of the network when viewed from terminals. A and B (ii) The equivalent resistance of the network when looked into from terminals A and 8, and (éii) Current in the load resistance Ry, of 152. [Ans. 1 Al 3Q A 3.692 6a =24V r=1Q 6 Fig. 94 Fig. 98 21, Find the current through 50 resistance in the circuit shown in Fig. 95. Use Thevenin's theorem. lAns. 0,086.A] 22. Calculate the current in the 8 9 resistor of Fig. 96 by using Thevenin’s theorem. (Ans. 0.8 Al Aa 4 8a 160 L 40 V— M M Fig. 96 Fig. 97 23. Uso Nortan’s theorem to calculate current flowing through 10 Q resistor of Fig. 97. 24, Fig. 98 shows network with the value of load resistance R;, = 50 ©. Develop Norton's equivalent circuit and determine the current and power delivered to Ry. Ans. 0.114.A ; 0.65 WI 8a 14.67 Fig. 98 Fig. 99 25, Fig. 99shows anumber of resistancesconnected in deltaand star. Find the resistance across the terminals Aand B. Use star/delta conversion method. [Ans. 2.96 2] D.C. CIRCUITS AND NETWORK ANAL’ a7 26. In the given circuit, find the resistance between the points A and C (Fig. 100). [Ans. 6.21 & 40.9 200 Toyy 65a Fig. 100 Fig. 101 27. By using delta/star transformation for Fig. 101, find the current J supplied by the battery. JAns. 0.95 Al 28, Find the current in the galvanemeter arm in tho network shown in tho Fig. 102, using deltatstar transformatian. Uns. 0.236 Al 1200 50.0 ev (A) ig. 102 29. A bridge network ABCD has arms AB, BC, CD and DA of resistances 2, 2, 4 and 2 ohms respectively. If the detector AChasa resistance of ohms, determine by star/deltatransformation, thonetworkresistance as viewed from the battery terminals BD. Ans. 4.35 01 A.C. Circuits 1. Introduction to alternating current. 2. Generation and equations of alternating voltages and currents. 3. Alternating voltage and current. 4, Single phase circuits—A.C. through pure ohmic resistance alone—A.C. through pure inductance alone—A.C. through pure capacitance alone— Phasor algebra—A.C. series circuit—R-L cireuit—R.C. circuit—R-L-C circuit—A.C. parallel circuits. 5. Transients—General aspects—D,C, transients—A.C. transients—Highlights— Objective Type Questions—Theoretical Questions—Unsolved Examplea, 1, INTRODUCTION TO ALTERNATING CURRENT A.C, means alternating current—The current or voltage which alternates its direction and magnitude every time, Now a days 95% of the total energy is produced, transmitted and distributed in A.C. supply. The reasons are the following : (é) More voltage can be generated (upto 39000 V) than D.C. (650 V only). (ii) A.C. voltage can be inereased and decreased with the help of a static machine called the ‘transformer’. (iii) A.C, transmission and distribution is more economical as line material (say copper) can be saved by transmitting power at higher voltage. (iv) A.C. motors for the same horse power as of D.C. motors are cheaper, lighter in weight, require less space and require lesser attention in operation and maintenance. (v) A.C. can be converted to D.C. (direct current) easily, when and where required but D.C. cannot be converted to A.C. so easily and it will not be economical. However, D.C. entails the following merits and hence finds wide applications. @) D.C. Aeris motors are most suitable for traction purposes in tramway, railways, crains and lifts. (ii) For electroplating, electrolytic and electrochemical processes (battery charging ete.), D.C. is required, (iii) Arc lamps for search lights and cinema projectors work on D.C. (iv) Arc welding is better than on A.C. (v) Relay and operating time switches, etc., and cireuit-breakers, D.C. works more efficiently. (vi) In rolling mills, paper mills, colliery winding, etc., where fine speed control of speeds in both directions is required, D.C. motors are required. 2. GENERATION AND EQUATIONS OF ALTERNATING VOLTAGES AND CURRENTS Generation of Alternating Voltages and Currents Alternating voltages may be generated in the following two ways : 1. By rotating a coil in a stationary magnetic field, as shown in Fig. 1. 2. By rotating a magnetic field within a stationary coil, as shown in Fig. 2. 48 AC. CIRCUITS 49 ‘Stator Fig. 1. Rotating a coil in a stationary Fig. 2. Rotating a magnetic field magnetic field. within a stationary coil. The value of the voltage generated in cach case depends upon the following factors : (i) The number of turns in the coils ; (ii) The strength of the field ; (iii} The speed at which the coil or magnetic field rotates. ® Out of the above two methods the rotating-field method is mostly used in practice. Equations of Alternating Voltages and Currents Fig. 3 shows a rectangular coil of N turns rotating clockwise with an angular velocity w radians per second in a uniform magnetic field. Since by Faraday’s law, the voltage is propor: tional to the rate at which the conductor its across the magnetic field or to the rate of change of flux link- ages, the shape of the wave of voltage applied to the external circuit will be determined by the flux distri- bution in the air gap. For a uniform field between the poles it is evident that maximum flux will link with the coil when its plane is in vertical position i.e., per- pendicular to the direction of flux between the poles. Also it is obvious that when the plane of coil is hori- t zontal no flux will tink with the coit. rr If the position of the coil with reference to the ‘ vertical axis be denoted by @ the flux linking with the Fig. 3. A coil rotating in a mngnet field. coil at any instant, as the coil rotates may be determined from the relation, 9 = Gnas 005 0 = Opugx COS WF adi) (Cs Ozu) where, 9,,,, = Maximum flux which can link with the coil, and t = Time taken by the coil to move through an angle 6 from vertical position. Using Faraday's law to eqn. (i), in order ta determine the voltage equation, dg ne dt (where e is the instantaneous value of the induced e.m.f.) e=- 50 ELECTRICAL TECHNOLOGY song g Pmas C08 Wh) = AN Cnas sin wt or e= oven sin @ alii) As the value of e will be maximum when sin 6 = ONG nar The eqn. (ii) can be written in simpler form as e=E,,,, 5in 8 . Alii) Similarly the equation of induced alternating current (instantaneous value) is t=J,_,3in 0 (if the load is resistive) ...(iv) Waveforms. A waveform (or wave-shape) is the shape of a curve obtained by plotting the instantaneous values of voltage or current as ordinate against time as abscissa. Fig. 4 (a, b, ¢, d, e) shows irregular waveforms, but each cycle of current/voltage is an exactly replica of the previous onc, Alternating ¢.m.fs and currents produced by machines usually both have positive and negative half waves, the same shape as shown. Fig. 4(/) represents a sine wave of A.C. This is the simplest possible waveform, and alternators are designed to give as nearly as possible a sine wave of e.m.f. ei ei ei 0 x 2R to n art m 2a t (a) (b) (c) ei ei ei 0 m 2n to r 2n eto 7 ory @ (e) cy) Fig. 4, Waveforms. e Ingeneral, an alternating current or voltage is one the circuit direction of which reverses at regularly recurring intervals. ® The waves deviating from the standard sine wave are termed as distoried waves. @ Complex waves are those which depart from the ideal sinusoidal form. All alternating complex waves, which are periodic and have equal positive and negative half cycles can be shown to be made up ofa number of pure sine waves, having different frequencies but all these frequencies are integral multiples of that of the lowest alternating wave, called the fundamental (or first harmonic). These waves of higher frequencies are called Aarmonics. AC. CIRCUITS 31 3. ALTERNATING VOLTAGE AND CURRENT Modern alternators produce an e.m.f. which is for all practical purposes sinusoidal (i.c., a sine curve), the equation between the e.m-f. and time being € = EB ya, Sin ot =A) where, ¢ = Instantaneous voltage ; E,.,, = Maximum voltage ; wt = Angle through which the armature has turned from neutral. Taking the frequency as f hertz (cycles per second), the value of w will be 2nf, so that. the equation reads e= Ey, 8in (np. The graph of the voltage will be as shown in Fig. 5. + Volts EMF. Pig. 5, The graph of the sinusoidal voltage. 1, Cycle. One complete set of positive and negative values of an alternating quantity is known as a cyele. A cycle may also sometimes be specified in terms of angular measure. In that case, one complete cycle is said to spread over 360° or 2n radians. 2, Amplitude. The maximum value, positive or negative, of an alternating quantity, is known. as its amplitude. 3. Frequency (f). The number of cycles/second is called the frequency of the alternating quantity. Its unit is hertz (Hz). 4, Time Period (T). The time taken by an alternating quantity to complete the cycle is called its time period. For example, a 50 hertz (Hz) alternating current has a time period of a second, Time period is reciprocal of frequency, ty Te $ (0 fe 3) nd 2) 5. Root mean square (R.MLS.) value. The r.m.s. (or effective) value of an alternating cur- rent is given by that steady (D.C,) current which when flowing through a given circuit for a given time produces the same heat as produced by the alternating current when flowing through the saine circuit for the same time. RM.S. value is the value which is taken for power purposes of any description. This value is obtained by finding the square root of the mean value of the squared ordinates for a cycle or half- cycle (See Fig. 5). This is the value which is used for all power, lighting and heating purposes, as in these cases the power is proportional to the square of the voltage. Refer Fig. 5. 52 ELECTRICAL TECHNOLOGY The equation of sinusoidal alternating current is given as : i=T,,,sin0 The mean of squares of the instantaneous values of current over half cycle is -f Pao “ta G=0) * * = 2f 2 do=2 Unax 8in 0)? d0 veh Pm sin’ ouo~ Hes [| fio se lao = [a- oneal 40 2n wo Eon go Lima to [ae st On 2 oe or 7=0,707 Leas -A3) Note. While solving problems, the values of given current and voltage should always be taken as the rans, values, unless indicated otherwise. 6, Average or mean value. The average value of an alternating current is expressed by that steady current which transfers across any circuit the same charge as is transferred by that alternat- ing current during the same time. The mean value is only of use in. connection with processes where the results depend on the current only, irrespective of the voltage, such as electroplating or battery charging. Refer Fig. 6. The valve of instantaneous current is given by isl,,,sin0 Refer Fig. 6. The value of instantaneous current is given by : ‘maz 81 0 [0 = av) bn as it? [ Limits are taken from 0 toz, since | only first half cycle is considered For whole cycle, the average value of sine wave is zero, . oa oe -sinodo=2, In| ~ om of 1 a a ee or 1,, = 0.637 I Ad) Note. In case of unsymmetrical alternating current viz. half-wave rectified current the average value must always be taken over the whole cycle. 7. Form and Peak Factors Form factor. The ratio of r.m.s. (or effective) value to average value is the form factor (K, ph of the wave form. It has use in voltage generation and instrument correction factors. AC. CIRCUITS 53 Peak factor. The ratio of maximum value to the r.m.s, value is the peak factor (K,) ofthe wave Average value = 2 bone #0637 I, 2 Half wave rectified sine wave: Nenae }—oworm — 4, RMS, value = a O05! Average Value = 2 Imax = 0.318 J... Full wave rectified sine wave : Neus (EVN __ a Fig. 9 E RMS. value = “92 = 0707 Tae Average value = 2 Ina = 0.637 Inns 8. No, Wave form Form factor (K)) Peak factor (K,) 2 ims value max.value average value s.vualue 1 Sine wave: mae Time ——» Fig. 7 Jn 0707 Ivar R.M.S. value = 70m qe K,= 0657 Ine sll 0.707 J, 54 ELECTRICAL TECHNOLOGY Form factor (K) Peak factor (K,) = eins walwe = max value ‘average value Fm.s, value RMS. value = Average value ‘Triangular wave : Joga Naas Fig. 11 Srna RMS. = = 0.5787 kB mar Tinax Average value = 3 705 Tea Reasons for using alternating current (or voltage) of sinusoidal form : An alternating current (or voltage) of sinusoidal form is normally used because of the following reasons : 1, Mathematically, it is quite simple. 2, Its integrals and differentials both are sinusoidal. 3. It lends itself to vector representation. 4, A complex waveform can be analysed into a series of sine waves of various frequencies, and each such component can be dealt with separately, 5. This waveform is desirable for power generation, transmission and utilisation. 8, Phase and phase angle. The ‘phase’ of an A.C. wave may be defined as its position with respect toa reference axis ar reference wave and ‘phase angle’ as the angle of lead or lag with respect to the reference axis or with respect to another wave. Examples. The phase of current at point L is < second where T is the time period or expressed in terms of angle 6, it is ; radian (Fig. 12). Similarly A.C. CIRCUITS 55 phase of the rotating coil at the instant shown in Fig. 13 is wt which is therefore called its phase angle. The e.m.fs. induced in both the coils (Fig. 13) will be of the same frequency and of sinusoidal shape, although the values of instantaneous e.m-f. induced will be different. However, the alternat- ing e.m.fs. would reach their maximum and zero values at the same time as shown in Fig. 13 (6). Such alternating voltages or curve are said to in phase with each other. 1 (Second) ——> (a) (b) Fig. 13 Refer Fig. 14. M lags behind L by B and N lags behind L by (a + B) because they reach their maximum later. Fig. 14 Example 1. (a) What is the equation of a 25 cycle current sine wave having r.m.s. value of 30 amps? (b) A 60 cycle engine-driven alternator has a speed of 1200 r.p.m. How many poles are there in the alternator ? 56 ELECTRICAL TECHNOLOGY Solution. We know that, i =/,,,, sin ot =I.,,, sin aft (: a@=onp . RMS, value_ 1 30 . (2n x 25 xt} : = 30x J2 . sin (2n x 28 x #! Maz vale +) = 42.42 sin 157 t. (Ans.) Using the relation, red ae where, f= frequency, N = speed in rpm, anid p=No. of poles co = ABP or p26. (Ans) Example 2. An alternating current varying sinusoidally with a frequency of 50 Hz has an rms. value of 40.4. Find : (i) The instantaneous value 0.0025 seconds after passing through maximum positive value, and (ii) The time measured from a maximum value when the instantaneous current is 14.14 A. Solution. Tnnay = V2 * 40 = 56.56 A w = 2nf = 2n x 50 = 1002 radians (i fel, cos wt after +ve maximum value = 56.56 cos 100 nt = 56.56 cos (100n x 0.0025) sé = 0.0025 5 ...given) = 56.56 cos 45° (Taking x = 180°) = 40 A. (Ans.) (ii) 14.14 = 56.56 cos (100 x 180 x t) 14.14 or jane 7° (100 x 180 « ¢) or cos”! (0.25) = 100 x 180 xt 75.5° = 100 x 180 xé t= 0.00419 s. (Ans.) Example 3. A sinusoidal alternating voltage of 50 Hz has an r.n.s. value of 200 V. Write down the equation for the instantaneous value and find this value 0.0125 sec. after passing through a positive maximum value. At what time measured from a positive maximum value will the instantane- ous voltage be 141.4 volts ? Solution. Refer Fig. 15. Vena = V2 x 200 = 282.2 volts w = Onf' = 2m x 50 = 100 7 rad/sec. Equation for the instantaneous voltage. V = Viage Sin cot (with reference to point O) = 282.2 sin 100 nt wel) Since the time (0.0125 sec.) is given from the point Z (i.e. from positive maximum value) the equa- tion @) when referred to point L can be written as v = 282.2 sin (90° + 100 nt) = 282.2 cos 100 nt AC. CIRCUITS a7 Hence instantaneous value of the voltage 0.0125 sec. after passing through + ve maximum. value, uv = 282.2 cos (100 x x 0.0125) = 282.2 cos (100 x 180 = 0.0125) (angle in radians) ingle in degrees) ° _i1 = 282.2 cos 226° 280.2 4) = 200 V (point M). (Ans.) Also v=M414V “ 141.4 = 282.2 cos (100 « 180 x t) 0.5 = cos (100 x 180 x t) cos! (0.5) = 100 x 180 x¢ 60* = 100 x 180 xt or te aw sec. (point N). (Ans.) Example 4. (a) What is the peak value of a sinusoidal alternating current of 4.78 rms. amps ? (b) What is the r.m.s. value of a rectangular wave with an amplitude of 9.87 volts ? (ec) What is the average value over half a cycle of a sinusoidal alternating current whose r.m.s. value is 31. A? Solution. (a) Peak value, J, = /2 x 4.78 = 6.76 A. (Ans.) (b) Refer Fig. 16, Ifthe first half-cycle is divided inton equal parts each of value V, then V=987V gas ve 2.ytyy? Tam.s. value = (ee n = V = 9.87 volts. (Ans.) e) 1. 231A i, fee Form factor "Fig. 16 = Gh =27.93 A. (Ans) Example 5. The graph in Fig. 17 shows the variation of voltage with time. Use the graph to calculate the average and r.m.s. value of the voltage. What is the frequency of the voltage ? What would be the r.m.s. value of sine wave having the same peak value ? Solution. Refer Fig. 17. As the graph is symmetrical about time axis, considering only the positive half cycle. Average value, Mago RTOS AG DS HDs 100 + 40 + 20 +10 = 46 V. (Ans.) RMS, value, Vx [07 +10" + 207 + 40° + 1007 + 1207 + 1007 + 40% + 207 + 107 . 10 58 ELECTRICAL TECHNOLOGY = fae = 3860 =62.1V. (Ans.) Sine wave T = 20 milli second Fig. 17 Since the time period ‘7” is 20 millisec. pod 1 Frequency v =F? doxi07 =50 He (Ans.) RMS. value of'a sine wave of the same peak value = 0.707 x 120 = 84.84 V. (Ans.) Example 6. Prove that ifa D.C. current Of Tops is superposed in a conductor by an A.C. current of max. value I amps, the r.m.s. value of the resultant is fit. Solution. Let the A.C. current be i = J sin 0 where i is the instantaneous value of the A.C. current and I the D.C. current. The r.m.s. value of U + i) over one complete cycle is, = 2 [rus Tsin 8)" d6 Qn _ fi pe . 2 =f (2+ 2sin 0+ sin“ 0) 1 pe — (1-con 20 ef te f s+aaine o(4=3822)} el |o-deosa+ 2. #in28 2 4 1 J (n-2+n+2) =f. Rl BiH ~ (Ans) nol 3] AC, CIRCUITS 39 Example 7. A resultant current wave is made up of tivo components : a 4A D.C. component and a 50 Hz A.C. component, which is of sinusoidal waveform and which has a maximum value of 4A. (i) Sketch the resultant wave. (ii) Write an analytical expression for the current wave, reckoning ¢ = 0 at a point where the A.C. component is at zero value and where di/dt is positive. il) What is the average value of the resultant current over a cycle ? (iv) What is the effective or r.m.s. value of the resultant current ? Solution. (i) Sketch of the resultant wave : The two current components and the result- ant current wave are shown in Fig. 18. (Ans.) (ii) Analytical expression. The instantane- ous value of the resultant current is given by &=(4+4 sin ot)=(4+4sin8). (Ans.) (iii) Average value. Since the average value of the alternating current over one complete cycle is zero, hence the average value of the resultant eur- rent is equal to the value of D.C. component ie., 4A (Ans.) (iv) Effective or r.m.s. value + @ = Brgy 819 cot Mean value of # over complete cycle is Fig. 18 1p e_ 1 pe we = de=— | (4+4sin 0) di tah "®8=E J (4 +4 sin 9)? do = af'os +32 sin 0+ 16 sin? 0) do = Ef [15-220 1s(2=$*22)|an Qn 2 =f tet asin 0-8 con 28) a0 an - a4 1 sin26 =f (210-a2e0s0-8x 2 i = 1 ((48x - 32 c0s 2x - 4 sin 4a) - (-92)] = 2% 04 4 on 2n RMS. value, I= /24 =4.9A. (Ans.) Example 8, Determine the average and effective values of the saw-tooth waveform shown in Fig. 19. (NUD ov ELECTRICAL TECHNOLOGY Solution. Refer Fig. 19. Since the voltage increases linearly, therefore, Vup= 229 eso. (Ans) The slope, for the internal 0 > is given by i= /,,,,. Remembering that SOLP is identical with MQN, RMS. value of current, Lene = Vr ais? af" Pde +f Baw | . (ep , 2 = (Oe 20 eae) | + Bae thm ELECTRICAL TECHNOLOGY 8 2 F (012.7 +0224, 7) = $06 F2,, =0.775 Igge (ADS) Average value of current, _ 1 STO | 7TI20 . 2 re 20 J, IT /20 a= Fal2h iit + on Ina te|=2 2 (Aim et Das [eal 2 prao 77/20 3/2) ef + Ina T ar j}2 7120 2| (20 tom) ary (2-2) 7/2 ar} 2\ 20) *!m«| a0 ~ 20 2| (202 iper Tr =— — |x —| — =_ . 29 sr )3(ae) * tas * Z| 1, = 3 oss Fae *T +02 Inge XT) 20,7 Lys (Ans) Example 11.A half wave single anode rectifier has a voltage given by 100 sin wt applied to it. Estimate the average value on the dc. side. Solution. The wave form on the d.c. side is as shown in Fig. 22. Mear/Average d.c. voltage will be = Weel 100 sin 8. 0 . ~ B[-coxo] ‘ \ bn or 2n Fig. 22 Example 12. Calculate from the first principles, the reading which would be indicated by a Aot-wire ammeter in a circuit whose current waveform is given by 10 sin wt + 3 sin Som +2 sin Sut. Solution, The expression for instantaneous current is : f= 10sin wt +3 sin ut + 2 sin Soot The hot-wire ammeter will read the “r.m.s. value” of the wave form. Now # = (10 sin at + 3 sin Sint + 2 sin Sut)? R.M.S. value of the current (wt) 1 pe , : in may)? im = Zi (110 sin of +3 sin Sut + 2 sin Sut)? dtwt)h 1 pe “(2h (10? sin® we + 3? sin? Sut + 2? sin? Git + 2 x 10 «3 sin eat sin Sut v2 +2510 2sinc sin Got +232 sin St sin Guede | AC. CIRCUITS. 63 af 10? (1~ cos Quxt) | 3? 2n 2 2 32 2 (1- cos Seat) +> (1— ee tomate a] *All the terms containing the product of two sines when integrated over the range 0 to 2x disappear. This is easily seen by splitting such terms into the difference of two cosines. Qn 2 2 = (56.5) = 7.62. (Ans.) Example 13. Four branches A, B, C, D in an AC. circuit meet at a junction point P. The currents in branches A, B, C flow towards P while the current in branch D flows away from P. The currents in branches A, B and C are i, = 20 sin 628¢ iy = 15 sin (628t — 1/6) i, = 25 sin (628t + n/3) Find an expression for the instantaneous value of current in branch D, and calculate (i) its frequency, and (ii) the heat (watts) that it would produce when flowing in a resistance of 5 ohms. Solution. Analytical method. Let the current flowing in the branch D be, ip = I, sin (628! + ) TL =D gpeay ©08 Oy + Tpipae C08 Vy + Deinas C08 O3 + Lies S08 > = 20 cos O° + 15 cos (— 1/6) + 25 cos 3 + Ip,,,, C08 @ B = 20+ 15 «52+ 2507 + fopge 0088 -(povee ae] ={satoeat aay 7 . rr re ee = 20 +19 + 12.5 +1p,,,, 008 0 = 45.5 +Ip,,,, CO8 0 EV SD gar SI Oy + Lamar SIM Oy + Temas BIN Oy + nme, SiN O = 20 sin 0° + 15 sin (- 1/6) + 25 sin W3+J,,_, sind w0- 15x 2425498 + Tina, BIN 5 — 75 + 21.65 + pgs SIO = 1AIS tog, SIN O Since all the currents are meeting at point P, * tH=0 ie, 45.5 + Iya, COS O=0 or Iya ¥ C08 O == 45.5 lid and iIv=s0 te, 14,15 + Tongx SIO = 0 OF Tyna * 811 = — 14.15 wld) From (é) and (i), Mpg, = (- 45.5)? + (- 14.157 = 47.6 A. @=tan" Ae = 197° or 3.44° Hence the current in branch D follows the relation, ip = 47.6 sin (678 ¢ + 3.44). (Ans.) w 6 (@ Frequency = on 2 = 100 Hz, (Ans.) 2 2 tin Heat produced = (aps) xR-(4] 5=5620W. (Ans) ot ELECTRICAL TECHNOLOGY 4. SINGLE PHASE CIRCUITS The study of circuits involves three basic types of units (R, L, C ie., resistance, reactance and capacitance respectively) and four possible series combination of them. The latter, in turn, may be arranged in many kinds of parallel, series-parallel, parallel-series or other complex circuits. 4.1, A.C. Through Pure Ohmic Resistance Alone The circuit containing a pure resistance R is shown in Fig, 23 (a). Let the applied voltage be given by the equation, veV,,,, sin@= V__, sin wt eli) —v i v Oran cot (a) (b) Fig. 23. A.C. through pure ohmic resistance alone. Then the instantaneous value of current flowing through the resistance A will be, —_— (ii) The value of current will be maximum when sin wt = 1 or (wt = 90") tna = Substituting this value in eqn. (ii), we get i= Iq, in wt Adit) Comparing (i) and (iii), we find that alternating voltage and current are in phase with each other as shown in Fig. 23 (b), also shown vectorially in Fig. 23 (ce). AC. CIRCUITS 6S Power. Refer Fig. 23 (ec) Instantaneous power, PEUES Vio, SIN cot XL, SIN UE Vie Eoaae SIN? tat = Mone tae 2 sin? ot = Yona toas (1 - cos 2a) Via J Vay J, = an oe - a . a Qaot 2 z 2 2 fos (Constant part) (Fluctuating part) V, 1, - For a complete cycle the average fe “Ee 08 2 wt is zero. Hence, power for the whole cycle, Pa Mo A6 = Vey lems or P=VI watt where V=R.MLS. value of applied voltage, and = RMS. value of the current. It may be observed from the Fig. 23 (c) that no part of the power cycle at any time becomes negative. In other words the power in a purely resistive circuit never becomes zero. Hence in pure resistive circuit we have : 1. Current is in phase with the voltage. vo 2. Current J = R where J and V are r.m.s. values of current and voltage. 3. Power in the circuit, P= VI = PR. 4.2, A.C. Through Pure Inductance Alone Fig. 24 (a) shows the circuit containing a pure inductance of L henry. Vv {2 V= Vinge SiN ot fa) Fig. 24 (a) Let the alternating voltage applied across the circuit be given by the equation, vsV,,,, sin ot wl) Whenever an alternating voltage is applied to a purely inductive coil, a back e.m.f. is pro- duced due to the self-inductance of the coil. This back e.m.f. opposes the rise or fall of the current through the coil. Since there is no ohmie drop in this case, therefore, the applied voltage has to overcome this induced e.m.f. only. Thus at every step, 66 ELECTRICAL TECHNOLOGY v= Vinge SIM ut, i pas (Sin at = 3) {e) Fig. 24 (bi, (c). A.C. through pure inductance alone. Resultant power zero. di vale . di or Vina Sin tw = D ae or di= Yow sin at dé Integrating both sides, we get faix [se sina ae ¥, wt) Vinge a: or j= Yann (- S580") _ Vou sin or - 2] ~_ Vinue gs [ -3] . or fey sin} of > waif) where X, = wL (opposition offered to the flow of alternating current by a pure inductances) and is called Inductive reactance. It is given in ohms if L is in henry and w is in radian/second. The value of current will be maximum when sin (we - 5) 21 V, Dinan = x, Substituting this value in eqn. (ii), we get i =Iyq,8in (on - 3) wilt) Power, Refer Fig. 24 (c) . . A n Instantaneous power, peuizV,,,, sinwt x1... in [ve - z) = = Vier fae B10 tat . COS (wt = ~ Yaar fines x 2 sin wt cos wt =i AE sin 2 AC. CIRCUITS 67 Power for the whole eycle, P= —Yingx !mx ("Sin ou = 0 ‘ower for the whole eycle, P= ~9g ee sin = Hence average power consumed in a pure inductive circuit is zero. Hence in a pure inductive circuit, we have : veoVv_eV 1. Current J = Ye = uL ” Saf amp. 2. Current always /ags behind the voltage by 90°. 3. Average power consumed is zero. t Variation of X, and f: x Since X, = wl = 2n/L, and here if L is constant, then Xf Fig. 25, shows the variation. As frequency is incrensed X,, i— increases and the current taken by the cireuit decreases. Fig. 25. Variation of X, with /. 4.3, A.C. Through Pure Capacitance Alone The circuit containing a pure capacitor of capacitance C farad is shown in Fig. 26 (a). Let the alternating voltage applied across the circuit be given by the equation, Ny V= Virus SIN cot 1 tas Bin (ut + 3) (b) Fig. 26. A.C. through pure capacitance alone. Resultant power is zera. v=V,,,, sin ox ol) Charge on the capacitor at any instant, q=Cu 68 ELECTRICAL TECHNOLOGY Current through the circuit, Ps —-=— (CV,,,. Sin wt) =o CV, cos ct or i= Fast sin (we +5) V, ™ " i= — sin ut+2) welt} Xe ( 2 1 The denominator X= Te (opposition offered to the low of alternating current by a pure capacitor) is known as capacitive reactance, It is given in ohms if C is in farad and w in radian/second. ‘The value of eurrent will be maximum when sin (ot + 2) 1 os x Xo Substituting this value in eqn. (ii), we get i= Tyg Si {ue + 2) Aid} Power. Refer Fig. 26 (c) Instantaneous power, t p=vi=Vy,,sinat x 1,,, sin (wt +2] V, L, = Var tongs 810 OF 608 Gat = FEE EE sin Quit 2k Power for the whole cycle = Noe ome sin 2at =0 This fact is graphically illustrated in Fig. 26 (c), It may be noted that, during the first quarter cycle, what so ever power or energy is supplied by the source is stored in the electric field set-up between the capacitor plates. During the next quarter cycle, the electric field collapses and the power or energy stored in the field is returned to the source. The process is repeated in each alterna- tion and this cireuit does not absorb any power. Hence in a pure capacitive eireuit, we have v L d= x = Vx 2 fC amps. c 2. Current always leads the applied voltage by 90°. 3. Power consumed is zero. Variation of X_ and f: t . 1 eat Xe Since X,= Re and if C is kept constant, than 1 Xn« = cme Fig. 27, shows the variation. As the frequency increases X;, decreases, so the current increases. Fig. 27 AC. CIRCUITS 69 4.4, Phasor Algebra The following are the methods of representing vector quantities : 1. Symbolie notation —_—-2. Trigonometrical form 3. Exponential form 4. Polar form. A veetor as shown in Fig. 28 may be described in the above forms as follows : 1. Symbolic notation + Exa+jb 2. Trigonometrical form : E= Ja? +? (cos 0+ sin 0) Fig. 28 = fa? +8? (cos 0 + sin 0) won general 3. Exponential form : E = fa? x6? ex = free & oowin general 4. Polar form : Ea fatse 20 seen 220 voucin general Significance of operator j. The letter j used in the above expressions is a symbol of an operation. It is used to indicate the counter-clockwise rotation of a vector through 90°. It is assigned a value of 1. The double operation of j on a vector rotates it counter-clockwise (CCW) through fen? =-1. In general, each successive multiplication of j, rotates the phasor further by 90° as given below (Refer Fig. 29) ‘gor j-fa «90° CCW rotation from OX-axis 2 180°} oO Fe(Fi) =-1 VeN +x +180" CCW rotation from OX-axis fs (WoT) =-fae-s 180° and hence reverses its sense because, j x j ..270" CCW rotation from OX-axis Pye niuye7er Fig. its ((Fi)' 41 em «860° CCW rotation from X-axis It should also be noted that, Lif si nL; jf =i” Example 14. Write the equivalent potential and polar forms of vector 6 + j8. Also illustrate the vector by means of diagram. mM Solution. Refer Fig. 30. 8 Magnitude of the vector = y6* +8 = 10, tand=5 Qs tan"(2)=sa 6 Exponential form = 10 0/1", (Ans.) The angle may also be expressed in radians. Polar form =10253.1%. (Ans.) The vector is illustrated by means of diagram in Fig. 30. ELECTRICAL TECHNOLOGY 6 Fig. 30 Example 15, A vector is represented by 30¢7"/4, Write down the various equivalent forms of the vector and illustrate by means of a vector diagram, the magnitude and position of the above vector. Solution, Refer Fig. 31. Draw the vector in a direction 2x 180 making an angle of 2. 3. 120° in the clockwise direction (since the angle is negative}. (i) Rectangular form : a= 30 cos (— 120°) =- 15 6 = 30 sin (= 120°) = = 25.98 Expressionis = (-15-j 26.98). (Ans.) (ii) Polar form =30 4-120". (Ans.) Addition and subtraction of vector quantities: For addition and subtraction of vector quantities rectangular form is best suited. Consider two voltage phasors represented as: Vy =a, + jb, and Vy =a, + jb, Addition. V = V, + Vy = (a, +jb,) + (a, +jb,) = la, +0) + (b, +.) ‘The magnitude of the resultant vector F = lay + ay)? + (b, +b The position of 7 with respect to X-axis is 0 = tan (ate ‘1 Subtraction. V = Vj, - ¥, = (a, +jb,) - (ay + jb.) = (a, - a) +b, — by) ‘The magnitude of the resultant vector F = lay — a2)? +0, — by? ‘Tho position of 7 with respect to X-axis is 0 = tan“! (a=& ay ~ Multiplication and division of vector quantities : It the vectors are represented in the polar exponential form, their multiplication and division becomes very easy and simple. Consider two voltage phasors represented as = 4 V, =a, +b, = V,20,, where @, = tan"? (4) H AC. CIRCUITS W = b, Vi, = 4, + jb, = V,20,, where 6, = tant (2) 2 Multiplication. When the phasor quantities are represented in polar form, while multiply- ing their magnitudes are multiplied and their angles added algebraically, ie, V, «= V,20, x V, 20, = V,V, 200, + 0,) Division. In this case, the magnitudes of phasor quantities (expressed in polar from) are divided and their angles subtracted algebraically . YM) fe. yo V,20, -y 410, - 0). Example 16. Perform tie following operation and express the final result in polar form : 10 230° + 16 Z= 30°. Solution. 10 £30° = 10 (cos 30° + j sin 30°) = 5.66 + j6 16Z-30° = 16 [cos (- 30°) + j sin (~ 30°)] = 13.86 -j8 10230° + 162- 30° = (8.66 + j5) + (18.86 -j8) = 22.52 - j3 = (¥2252" +3? jtan! (— 3/22.52) = 22.72 tan! (— 3/22.52) = 22.72 2-7.6". (Ans.) Example 17. Subtract the following given vectors from one another. A = 15 +j26 and B =- 19.75 -j7.18. Solution. A ~ B = € =(15 + 26) —(— 19.75 —j7.18) = 84.75 + j33.18 Magnitude of & = [34.757 + 33.187 = 48 Slope of © = tan~! (89,18/34,75) = 43,68" C = 48 243.68". (Ans.) AB Example 18. Perform the operation = and express the final result in polar form for the vectors given below : A =10+j10; B = 152- 120°; © =5+j0°. Solution. Rearranging vectors A and in polar form, we have A =10+j10 = J107 + 10% tan! (10/10) = 14.14 248" © =5 +0 = 5? + oF tan” (0/5) = 520" AB 14)4. 246° x 152-120" _ 14.14 x 16 € * 520° 5 =42.42/-75". (Ans.) Example 19. The instantaneous values of two currents i, and ig are given as : 2(45° - 120° -0°) i, = Sin (+4) and iy = 2.5 cos (wt - 2) Find the r.m.s. value of i, + ig using complex number representation. (Kerala University) 72 ELECTRICAL TECHNOLOGY Solution. Given: i, = 5 sin (w + *), iy =2.5 cos (wr- 2} =25 sin [20 +(w-2)] = 25 sin ot Icon) = 5 (C08 45° + j sin 45° Janagy = 2-5 (€08 0° + sin 0" The maximum value of resultant current is Tinag = (3.53 + j3.53) + (2.5 + j0) = 6.03 + 3.53 = 6.987 230.34° 6.987 RMS. value = “E* 4.94 A. (Ans.) Conjugate complex numbers ‘Two numbers are said to be conjugate if they differ only in the algebraic sign of their quadrature components. Accordingly, the numbers (a + jb) and (a — jb) are conjugate. @ The sum of two conjugate numbers gives in-phase or active component and their difference gives quadrature or reactive component. ie (a +jb) +(a—jb)=2a — (ie., active component), and (a +jb) -(a—jb)=j2b (i.e., reactive component). The resultant is the sum of two vertical components only. ® The resultant arising out of the multiplication of two conjugate numbers contains ne quadrature component. ie, (a+ jb) x (a — jb) = a? = 728? = a? + b? The conjugate of a complex number is used to determine the apparent power of an A.C. circuit in complex form. Power and roots of vectors/phasors The powers and roots of vectors can be found conveniently in polar form. If the vector are not, in polar form, these should be converted into polar form before carrying out the algebraic operations, as mentioned below. Powers. Consider a vector phasor quantity represented in polar formas A =A20, Then (Ay = A*A(n x 0) Example. Suppose it is required to find cube of the vector 4.212” Then, (4212°8 = (4)8 2 (3 « 12°) = 64 236° Roots. Consider a vector (phasor) quantity represented in polar form as (A )“* = (A)* zon. Example. Suppose it is required to find eube root of 125 260° Then, (125 260°)" = (125)"* 260°/3 = 5220°. The 120° operator av —~ In case of 3-phase work, where voltage vectors are displaced by 120° from one another (Fig. 32) it is convenient to use an operator, See) which rotates a vector/phasor through 120° toward or backwards with- “0° out altering its length. This operator is ‘a’ and any operator which is 2 multiplied by ‘a’ remains unaltered in magnitude but is rotated in CCW (counter-clockwise) direction by 120°. cs a=12120° av In cartesian form, a= cos 120° +j sin 120° =-0.5 +) 0.866 AC. CIRCUITS 3 Similarly, a? = 1 2120° x 1 2120" = 1 2240° = cos 240° +f sin 240° = - 0.5 -—j 0.866 Hence the operator ‘a’ will rotate the in CCW by 240° which is the same as rotating the vector in CWielock-wise) direction by 120°. os at=12- 120° Similarly a! = 12360" =1 (Numerically, a is equivalent to the cube root of unity.) 4.5, A.C. Series Circuits Under this heading we shall discuss R-L, R-C and R-L-C series circuits. 4.5.1. R-L circuit (Resistance and inductance in series) Fig. 39 (a) shows a pure resistance R and a pure inductive coil of inductance L connected in series. Such a circuit is known as R-L circuit (usually met a cross in practice). V_= IR A (0) Phasor diagram {I lags V by angle o) (a) Circuit diagram c v= Vas Sin cat y 1 = las (Sin tot -9) 2 Xsok t A (@) Resolution of | () Power curves Fig. 33. R-L circuit (Resistance and inductance in series). 74 ELECTRICAL TECHNOLOGY Let V=R.MS. value of the applied voltage, J = RMS. value of the resultant current, V_ = IR = Voltage drop across Rin phase with 2), and V,, = IX, = Voltage drop across Z (coil), ahead of I by 90°. The voltage drop V, and ¥, and shown in voltage triangle OAB in Fig. 33 (6), / being taken as the reference vector in the phasor diagram. Vector OA represents ohmic drop V, and AB represents inductive drop V,. Vector OB represents the applied voltage V which is the vector sum of the two (ie, Vpand V,). Va Vg? + V2 = fURy + UX, )? = HR? + X,2) I Vv Vv or = =v Wevaxy # where Z = JR? +X," (total opposition offered to the flow of alternating current by R-L series cir- cuit) is known as impedance of the circuit. As seen from the “impedance triangle” ABC (Fig. 33 (ec), Bs R+X,? ie., (impedance) = (Resistance) + (Inductive reactance)” From Fig. 33 (b) it is evident that voltage V leads the current by an angle @ such that, tang = Ve = Xt Xt, 9h _ Inductive reactance Vp IR ROR Resistance . gstar (%2) The same is illustrated graphically in Fig. 33 (d). In other words J Jags V by an angle 6. R . Power factor, cos 9 = Zz (From Fig. 33 (c)] ‘Thus, if the applied voltage is given by u = V,,,, sin wt, then current equation is given as, i=J,,,, Sin (@t-9), where Jovan = max In the Fig. 33 (e), [ has been shown resolved into two components, J cos 6 along V and J sing in quadrature (/.e,, perpendicular) with V, Mean power consumed by the cireuit = Vx J cos 9 (ie., companent of I which is in phase with V he, P=VIcos >(=r.m-s. voltage x r.m.s, current * cos 6) ‘The term ‘cos ¢' is called the power factor (- 3) of the cireuit It may noted that : —In A.C. circuit the product of r.m.s. volts and r.m.s. amperes gives volt-amperes (i.e, WA) and not true power in watts. True power (W) = volt-amperes (VA) = power factor or Watts = VA (Apparent power) x cos —The power consumed is due to ohmic resistance only since pure inductance consumes no power. AC. CIRCUITS 75 ie, PeVicog=VIx =< i= 1x IR = PR, watts vo Gr 03.9 = RIZ and = 1) This shows that power is actually consumed in resistance only ; the inductor does not consume any power, The power consumed in R-L circuit is shown graphically in Fig. 33 (). Thus in R-L circuit we have : 1. Impedance, Z = [R? +X," (where X, = ob = 2n x fL) Vv 2. Current, P= > zZ R True power _ W 3. Power factor, cos ¢ = z( SEAS AL lor angle of lag, @ = cos“ (R/Z)) 4 Power consumed, P = VIeos@(=12 x12 <1" R) Symbolic Notation : ZaR+jX, The numerical value of impedance vector = JR” +X; The phase angle with the reference axis, 9 = tan (X,/R). In polar form : B=220°. Apparent, Active (True or real) and Reactive Power: Every circuit current has two components : (i) Active component and (ii) Reactive component. “Active component” consumes power in the circuit while “reactive component” is responsible for the field which lags or leads the main current from the voltage. In Fig. 34. active component is /,,,,,. = cos >, and reactive componentis J,,,..:,.=/ sind Icos o Rinneceeneereeseneeneene Ising ' Fig. 34. Active and reactive components Fig. 35, Apparent, true and reactive power. of circuit current I. A Fective)® + Tpeative ™ So, 76 ELECTRICAL TECHNOLOGY Refer Fig. 35. . (2) Apparent power (S). It is given by the product of r.m.s. values of applied voltage and circuit current. S=VJ=(1xZ).1=/Z volt-amperes (VA) tii) Active or true or real power (P or W). [tis the power which is actually dissipated in the circuit resistance. P= PR =VI cos 6 watts (itt) Reactive power (Q). A pure inductor and a pure capaciter do not consume any power, since in a half cycle what so ever power is received from the source by these components the same is returned to the source. This power which flows back and forth (é.e., in both directions in the circuit) or reacts upon itself is called “reactive power.” It may be noted that thecurrent in phase with the voltage produces active or true or real power while the current 90° out of phase with the voltage contributes to reactive power. In a R-Lcireuit , reactive power which is the power developed in the inductive reactance of the circuit, is given as: Q=PX,=PZsing=1,UZ)sine = VI sin 9 volt-amperes-reactive (VAR) These three powers are shown in Fig. 35 Relation between VA, W and VR W= VA cos sli) VAR = VA sing wei) “ VA= + + [From (i)] 05 9 VAR a and, VAs ane +» [From (ii)] W __ True power 7 or Apparent power ‘The larger bigger units of apparent, true and reactive power are kVA (or MVA), AWor MW) and kVAR (or MVAR) respectively. The power factor depends on the reactive power component. If it is made equal to the active power component, the power factor becomes unity. Example 20. A coil takes 2.5 amps. when connected across 200 volt 50 Hz mains. The power consumed by the coil is found to be 400 watts. Find the inductance and the power factor of the coil. Solution. Current taken by the coil, 7=2.5 A Applied voltage, V = 200 volts Power consumed, P=400W We know that P=VIcosd Power factor (p.f.) or 400 = 200 x25 x 0089 or cos d= SOO 208 Hence power factor of coil is 0.8, (Ans.) Imped: fthecoil, Z ¥.m™ 802 pedance of the coil, =T"tet Also 3b sine X,=Zsino AC. CIRCUITS 7 = 80 sing = 80 J1- cos! 9 8" = 80 x 0.6 = 48.2 But X, = 2 p= Si =_48_ . 0.1590 H (henry). (Ans.) anf = 2nx50 Example 21. A 100 V, 80 W lamp is to be operated on 230 volts, 50 Hz A.C. supply. Calculate the inductance of the choke required to be connected in series with lamp for its operation. The lamp can be taken as equivalent to a non inductive resistance. Solution. Current through the lamp when connected across 100 V supply, Ww 80 Ts Vv" I00 798 A Resistance of the lamp, R= ; = 22 a r250 If a choke of inductance L henry is connected in series with the lamp to operate it on 230 V, the current through the choke will also be 0.8 A. The impedance of the circuit when choke is connected in series with the lamp, V _ 230 Z= T"t87 287.52 Reactance of choke coil, X, = \Z? - R? = 287.5? - 125? = 258.59 X,, = 2afL But or Le Xp, . 2585 Hence inductance of choke coil, L = 0.825 H. (Ans.) Example 22. A coil has a resistance of 5 Q and an inductance of 3.8 mH. Calculate the current taken by the coil and power factor when connected to 200 V, 50 Hz supply. Draw the vector diagram. If a non-inductive resistance of 10 Q is then connected in series with coil, calculate the new value of current and its power factor. Solution. R52 L = 31.8 mH or 0.0318 H X, = 2nfL = 2n x 50 « 0.0318 = 102 R(=5a) L (231.8 mH) aL eo “ Va= IR I 200V, 50 Hz (a) FAL circuit {b) Vecter/phasor diagram Fig. 36 78 ELECTRICAL TECHNOLOGY Impedance of the coil, Za 4R?+X,? = ¥5? +10? = 11.182 = 17.9 A. (Ans.) ,_¥_ 200 . Current taken by the coil, T= >= T7g R Power factor, cose= F= Fig. 36 (b) shows the vector diagram. When non-inductive resistance of 10 Qis connected in series with the coil : Total resistance in the circuit, R’=5 + 10= 152 Reactance inthe circuit, -X,' =X, = 102 Impedance of the circuit, aa fr? +x’? = S18? +10? - 189 Vv _ 200 Current through the cireuit, =’ = >; =—— = 11.11 A. (Ans.) “2 18 Power factor of the circuit, cos 9 = z = z = 0.833. (Ans.) Example 23. A current of 5A flows through a non-inductive resistance in series with a choking coil when supplied at 250 V, 50 Hz, If the voltuge across the resistance is 125 V and across the coil 200 V, cateulate : (i) Impedance, reactance and resistance of the coil, (ti) The power absorbed by the coil, (iif) The total power, Draw the vector diagram. (Elect. Engg. Madras Univ.) Solution. Non-inductive resistance connected in series with coil = 15 =252 5 Refer Fig. 37 (h). Cai 30 8 __ A__ 5A 4 x 125 V- 200 250 V, 50 Hz (a) Series circuit Fig. 37 BC? + CD? = (200)? = 40000 oli} (125 + BC? + CD® = (250)? = 62500 Aid) Subtracting eqn. (é) from eqn. (ii), we get (125 + BC)? - BC? = 62500 - 40000 = 22500 15625 + BC? + 250BC — BC? = 22500 BC = 27.5V; CD = i200)" - 275)" = 198.1 V AC. CIRCUITS: 9 200 (i) Coil impedance, Zz >? 409. (Ans.) V,_=IR=BC = 27.5 or 5R = 27.5 R= 723 2550 (Ans) Also, V, = IX, = CD = 198.1 198.1 x, “8! -ane20. (Ans.) (i) Power absorbed by the coil, P=PR=5?«55=137.5 W. (Ans.) (iii) The total power = Vicos 9 = 250.x 5 x AC 7. = 250 «5 x Meee = 762.5 W. (Ans.) The vector diagram is shown in Fig. 37(8). (Ans.) Example 24. An iron-cored coil has a D.C. resistance of 6 ohms. When if is connected to 230 V, 50 Hz mains, the current taken is 3.5 A at a power factor of 0.5. Determine : (i) Effective resistance of the coil. (ti) Inductance of the coil. (iii) Resistance which represents the effect of the iron loss. Solution. Given : D.C. resistance (True resistance), R = 6 2 ; supply voltage = 230 V, f= 50 Hz, J=35A; pf = 05. (i) Effective resistance of the coil, R,: ‘Total power consumed by the iron-cored choke coil, a Powe loss in ohmic resistance + Iron loss in core = PR + P, or =R+4 z FP where f is known as effective resistance of the coil. P_Vicos® 230*3.5x05 Effective resistance, R, = Rp + aF = 32.86 2. (Ans.) (i) Inductance of the coil, L: Impedance of the coil, = ts = =65.72 Inductive reactance of the coil, X,= (2*_R? =Jt5.777 - 3286)" =56.92 X, __ 569 Loy Qnx5o = 021811 H. (Ans.) Gii) Resistance representing iron loss: Since Saree Effective resistance, R, = True resistance + Resistance representing iron loss 92.86 = 6 + Resistance representing iron loss Resistance representing iron loss = 32.86-6 = 26.86. (Ans.) 8a ELECTRICAL TECHNOLOGY Example 25. Three coils connected in series across a 100 V, 50 Hz supply have the following parameters ; R,= 180, L, = 0.012 H; Ry = 12 Q, Ly = 0.036 H; Determine the potential drop and phase angle for each coil. Solution. Fig. 38. shows the cireuit diagram. Total resistance in the circuit, R= R, +Ry+Ry = 18+124+3.6=33.62 Total inductance in the circuit, L = L, +L, + Ly = 0.012 + 0.036 + 0.072 = 0.12H Impedance of coil-1, Z, = [Ry + (2nf Ly) = JS) + (an x50 x 0.012)" = 18.390 Impedance of coil-2, Z,= (Rj! + (2x fylg?* = (2) +(2n x 50x 0.036)" = 16.49 2 Coil-1 Coill-2 Coil-3 n_ = 3.69, L,= 0.072 H a 180 0.012H 120 0.036 H 369 0.072H 100 V, 50 Hz Fig. 38. Circuit diagram. Impedance of coil-3, Z, = y/Ry? + (2n f Ly)? = a6)? + (2n x 50 x 0.072)" = 22.902 Impedance of the whole circuit, Z = JR? + (anf LY = (33.6)? +(2n x50 x 0.12)" = 50.5.9 ¥ 300 sos Potential drop across coil-1, V, = /Z, = 1.98 x 18.39 = 36.41 V (Ans.) Potential drop across coil-2, V, = IZ, = 1.98 x 16.49 = 32.65 V (Ans.) Potential drop across coil-3, Vy = 72, = 1.98 x 22.90 = 45.34 V (Ans.) Phase angle of coil-1, 9,, cos“! (R/Z,) = cos“! (18/1839) = 11.82". (Ans.) Phase angle of cnil-2, 9, = cos“! (RYZ,) = cos-412/16.49) = 43,3", (Ans.) Phase angle of coil-3, @, = cos“! (Ry/Z,) = cos“43.6/22.90) = 80.96". (Ans.) Example 26, An alternating voltage of (176 + j132) is applied to a circuit and the current in the circuit is given by (6.6 + j 8.8) A. Determine : (i) Values of elements of the circuit. (ii) Power factor of the circuit. (iii) Power consumed, Solution. Given : Supply voltage, ¥ = 176 +132 = 220 236.87" Circuit current, J = 6.6 +/ 6.8 = 11 253.13° Current through the circuit, J = AC, CIRCUITS 81 (i) Values of elements of the circuit, R, C : ae = ¥ , mocear . . Circuit impedance, Zowe Ties = 20 4-16.26" = % [cos (= 16.26*) + sin (— 16.26°)] = 20 (0.96 - j 0,28) = (19.2 - 5.6) 2 R=19.29 (Ans.) Kon 880 C= A= Sayre F=5684 pF. (Ans. (ii) Power factor of the circuit ; cos 9: cosg = = 222 _ 9.96 (leading). (Ans.) Zz 2 (iit) Power (true) consumed, P': Apparent power, S2Vxi = 220 236.87* x 11 2-63.13° = 2420 2- 16.26° = 2323.2 -7 677.6 ‘True power, P=2323.2W. (Ans.) (Alternatively : P = VI cos ¢ = 220 x 11 x 0.96 = 2323.2 W). Example 27. in a circuit, the equations for instantaneous voltage and current are given by, vsI4id4sin (ue 24), volt and i= 7.07 sin («-§). amp, where w = 314 rad{sec. (i) Sketch a neat phasor diagram for the circuit (ii) Use polar notation to catculate impedance with phase angle. (iii) Calculate average power. (iv) Calculate the instantaneous power af the instant t = 0 (Pune University) Solution. Given : v = 141.4 sin (me - =), and i = 7.07 sin (we |, where w = 314 rad/s. (@) Phasor diagram : From the voltage equation, it is seen that the voltage lags behind the reference quantity by 180 180 = rad or 2x = = 120", Similarly, current lags behind the reference quantity by = rad or =~ = 90°. Between themselves, voltage lags behind the current by (120°-90°) = 30° as shown in Fig. 39 (6). (ii) Impedance with puase angle (polar notation) Ve Tage = = =100V; I= A - ue =5A. ce V= 100 £- 120° and / = 5 2-90° : z_¥ _ 1002-120 ~ ro 62-90° =20 2-30°9 (Ans) 82 ELECTRICAL TECHNOLOGY itl fa) (b) . Fig. 39 (iii) Average power: Average power = VI cos @ = 100 x5 x cos 30°=433 W. (Ans.) (iv) Instantaneous power att =0: At ¢=0;0= 141.4 sin (0- 120°) =- 122.45 V i= 7.07 sin (0 - 90°} =-7.07A Instantaneous power at t = 0, psu, =(~ 122.46) x (— 7.07) = 865.7 W. (Ans.) Example 28. A voltage e(t) = 100 sin 314 ¢ is applied to a series circuit consisting of 10 ohms resistance, 0.0318 henry inductance and a capacitor of 63.6 pF. Determine : (i) Expression for ift). (ii) Phase angle between voltage and current. (iii) Power factor. (tv) Active power consumed. (v) Peak value of pulsating energy. (Indore University) Soution. Given ; e(¢) = 100 sin 314 t, R = 10 Q, L = 0.0318 H, C = 63.6 uF = 63.6 x 107 F. A, CIRCUITS 83 1 1 He = 914 rad/s ; X, = wh = 314 « 0.0318 = 10 2; X. = — = ——__—_z = 509; re TOG * C™ wl 314 x 686% 10 X=X,-X,,= 10 - 50 = - 40 Q (capacitive) = . 7 ¥ _ aoon2) 20 = 76°; | 2s Z =10-j40 241.2 2-76": 1 = ae Tinag 2X V2 = 1.716 x JB = 243A (i) Expression for i(t) : i(t) = 2.43 sin (314+ 76°). (Ans.) (i) Phase angle between voltage and current, ¢: = 76° with current leading. (ii) Power factor, cos o: cos. @ = cos 76° = 0.24 (lead). (Ans,) (iv) Active power consumed, P : Ps VI cos @ = (100//2) (2.43//2) x 0.24 = 29.16 W. (Ans.) (iv} Peak value of pulsating energy : Refer. Fig. 40, The peak value of pulsating energy = Yasha + Voes ts con . Mae tse (14 cos 9)= O22 (1. 0.24) = 150.66 W. (Ans) 4.5.2. R-C circuit (Resistance and capacitance in series) Fig. 41, (a) shows a pure resistance R (ohms) and a pure capacitor of capacitance C (farads) connected in series. Such a cireuit is known as R-C circuit. Let, V= R.M.S. value of the applied voltage, T=R.M5. value of the resultant current, Vy = JR = Voltage drop across R (in phase with J) and Vo = IX, = Voltage drop across C, lagging J by 90°. . I = [3 = L716 276° a c Vp = IR—sf4e—Ve = [Xe v V= Vinax Sin cat (a) Circuit diagram (b) Phasor diagram (Ileads V by angle o) Zz Xe c (c) Impedance triangle (a) Power curve Pig. 41. R-C circuit (Resistance and capacitance in series). Voltage drops V, and V, are shown in voltage triangle OAB in Fig. 41 (4) being taken as the reference vector in the phasor diagram. Vector OA represents ohmic drop V, and AB represents the capacitive drop V,. Vector OB represents the applied voltage V, which is the vector sum of the V, and ¥,} Ve gt + Vo? = (UR +X, F <1 [+ x? v v Is => “ Ve +X7 2 where Z= {R? + X¢* (total opposition offered to the flow of alternating current by RC series circuit) is known as the impedance of the circuit. As seen from the “impedance triangle” ABC (Fig. 41 (c\, BoR+X2 ie. (Impedance)? = (Resistance)? + (Capacitive reactance” From Fig, 41 (6) it is evident that J leads the voltage V by an angle @ such that, tane= Me _ Xe 2 Xe, Wot) _ Capacitive reactance Ve IR R R Resistance X ostan? (4¢) The same is illustrated graphically in Fig. 41 (a), In other words I leads V, by an angle 9. Power factor, cos 9 = 2 {From Fig. 41 (c)) Power. Refer Fig. 41 (d), Instantaneous power, p=vi=V,,,, sin wt x J... sin (wt + @) = Yards x 2sin (wt + 0) sin wt ¥, = Sap x fa [cos o — cos (2 wt + @)) Average power consumed in the circuit over a complete cycle, P= Average of “age, fae cos 9 - Average of Mage tage cos (2uit +) AC CIRCUITS 85. or : P= Nag Heat cos ¢~ ser, or PHV Ls ¥ 1m, 6080 = VI coso where cos 9 is the power factor of the circuit: Alternatively, P=VIcos9= IZ x1 2 =PR ‘This shows that power is actually consumed in resistance only ; the capacitor does not con- sume any power. . Thus in R-C circuit, we have: 1, Impedance, Z = JR? + X¢” (where Xce = . avetiameel, * Vv 2. Current, / = Z 3. Power factor, cas 9 = Zz = for angle of lead, = cos“ (R/Z)) 4, Power consumed, P = VI cos 6 (= PR), Example 29. A capacitance of 20 uF and a resistance of 100 ohms are connected in series across 120 V, 60 Hz mains. Determine the average power expended in the circuit. Also draw the vector diagram. Solution. R=100Q C = 20 pF = 20 x 10- F (farad) R=1009 C=20"F z( ‘True power =| 120 V, 60 Hz (a) R-C circuit (0) Vector/phasor diagram Fig. 42 Capacitive reactance, Xo 1. 1 1592 . Sor "SocanD enn = pa Co BnfC In x 50x 20 x 10 Impedance of the circuit, Zs {R? +X¢? = 100" + 150 = 188 9 es Vv 120 Current through th t T= =~ =0.638A surrent through the circuit Z * iss R_ 100 Pe ctor, == 2 0.532 ‘ower factor, c= = Fa5 * Average power expended in the circuit, P,, = VIcos 0 = 120 x 0.638 x 0.632 = 40.75 W. (Ans.) Fig. 42 (b) shows the vector/phasor diagram. 86 ELECTRICAL TECHNOLOGY Example 30. A voltage v = 100 sin 314t - 50 cos 314t, is applied to a circuit having R = 20Q in series with C = 100 uF. Obtain expression for instantaneous current, r.in.s. value of current and the power in the circuit, (PTU, 1999) Solution. Given : v = 100 sin 314¢ - 50 cos 314¢; R= 202; C = 100 pF. ‘The R-C circuit and the phasor diagram for the given instantaneous voltage are shown in Figs. 43 and 44 respectively, 20a 100 pF fp c v= 100 sin 3141 50 cos 3141 Fig. 43 Fig. 44 Resultant voltage, Vise Phase angle with the horizontal, @ = tan (- 2) =~ 26.56° v= 1118 sin (314 — 26.56°) pe ® ¥(100)? + (60)? = 111.8 V Now, ov = 314 torm=314 x.-t. 1 c™ wl 314 100 x 10° Z= JR? + X¢? = yoo)" + 31.85" =37.60 ~. Capacitive reactance, =31852 . ¥, 111.8 Maximum value of current, Ipoe = =" 37.6 =297A Phase difference between voltage and current, R 20 1 = cos? (2) = cos"? | ——— | = §7.86° (leading) Zz 376 8 «. Instantaneous value of current i = 2.97 sin (314 - 26.56° + 57.86°) = 2.97 sin (3142 + 31.3°) or i= 2.97 [sin 314¢ . cos 31.8° + cos 314¢ , sin 31.3°) or i= 2.64 sin 914 + 1.64 cos 314t. (Ans.) Inax _ 2.97 RMS value of the current, Lm “HE= eB A. (Ans) Power in the circuit, P=Vicoso V,, = EE ates > x 2.1 x cos (57.86*) = 88.32 W (Ans.) Example 31. A two element series circuit is connected across an A.C. source ¢ = 200V2 sin (ut + 20°) V. The current in the circuit then is found to be i = 1002 cos (314 t - 25°) A. Determine parameters of the circuit. (Allahabad University) Solution, Given : e = 20/2 ain (we + 20°) i= 102 cos (3141 - 25°) AC. CIRCUITS 87 Parameters of the circuit, cos ¢ R,X, and C: The current i can be written as, i= 10/2 sin (914 ¢ - 25° + 90°) = 19/2 sin (314 ¢ + 65°) It is seen that applied voltage leads by 20° and current leads by 65° with regards to the reference quantity, their mutual difference is 65° - 20° = 45°. Henee, pf cos} = cos 45° = 0.707 (lead). (Ans.) Now, Vinax = 200V2 and I... = 10/2 Vonae _ 20008 Zim News - =202 ""Tnar OVE R= Zcos9=20% 0.707 = 14.140. (Ans.) X_.2Zsin @= 20% 0.707= 14,149. (Ans.) 14 » Xo= =o = i Also, cag ie where fe as = 50H: Xp= M4 = ae 1 = Tpqaxaangg P2261 MF. tans.) Hence the given circuit is an R-C cireuit. 4.5.3. R-L-C circuit (Resistance, inductance and capacitance in series) Fig. 45 shows a R-L-C circuit. or c Phasor dlagram Impedance triangle (e) Xe > Fig. 45. Resistance, inductance and capacitance in series. 88 ELECTRICAL TECHNOLOGY Important formulae: 1. Impedance, Z= yr? +(X,- Xo [whore X,, = 2nfl, Lin henry and Xp == 4, Cin tral 2. Current, Is zNI< 3. Power factor, cos O = Zz [ange of lag (when X;, > X¢) or lead (when X¢ > X;,), @=cos™? 4 4. Power consumed = Vi cos (= FPR) Resonance in R-L-C circuits Refer Fig. 45 (a). ‘Current {() —> "Frequency ()——> Fig. 46. Reactance (X) ws frequency (f). Fig. 47. Current in R-L-C circuit ws frequency. The frequency of the voltage which gives the maximum value of the current in the circuit is called resonant frequency, and the circuit is said to be resonant, At resonance, Xp =Xp (ie, Z=R) 1 6e.5 2Qa6.L = — te Nha TEE 1 f= LO (5) where f, = Resonance frequency in Hz; L = Inductance in henry ; and C = Capacitance in farad. Fig. 46 shows variation of X,, X, and X (total reactance = X, - X,) with variation of frequency f. Fig. 47 shows the variation of current (J) with frequency (/). At series resonance, it is seen that: 1, Net reactance of the circuit is zero ie, X, -X_- = 0 or X = 0. 2. The impedance of the circuit is minimum and equal to the resistance (R) of the circuit . Vv (ise Is ¥). Consequently cirewit admittance is maximum. 3. The current drawn is maximum (ie., J, = I,,..). 4. The phase angle between the current and voltage is zero ; the power factor is unity. AC. CIRCUITS 89 1 tlie ' if the frequency is below the resonant frequency the net reactance in the circuit is capacitive and if the frequency is above the resonant frequency, the net reactance in the circuit is inductive. 6. Although V, = V;, yet V,,, is greater than V_ because of its resistance. Half power frequencies, Bandwidth and Selectivity Half power (cut-off) frequencies : The half power frequencies are those frequen- cies at which the power dissipation in the circuit is half of the power dissipation at resonant frequency f,. They are the corresponding frequencies f, and /, at the value of current / = [./,J2 ; wherel, is the current at resonance in R-L-C series circuit (Refer Fig. 48). Hence power, P, drawn by the circuit at the resonance is it 5. The resonant frequency is given by f. = ty P.=I2R (6) Frequency 2 . Fig. 48 Power in the cireuit at f, = (4) ReZIeR ‘% (= half the power at resonance) 2 Power in the circuit at f, = ( 4 ) Re pur (= half the power at resonance) & Rk Also, Aeh- roa Bandwidth and Selectivity : The difference (f, - /,) is called the bandwidth (B,,) of the resonant network. The ratio of the bandwidth to the resonance frequency is defined as the selectivity of the circuit. When frequency is varied in R-L-C circuit, the selectivity becomes “h-f 1 “Bef “3 tn . where Q, is the quality factor of the resonant circuit. *Relation between bandwidth and quality factor in series resonant conditions : A series R-L-C circuit is considered. The resonant frequency and angular frequency are ex- pressed by /, and w, respectively. In the above circuit, the current (2) can be described as follows : v T= ee where V, 2, Land C are the source voltage, resistance, inductance and capacitance of the circuit respectively. 90 ELECTRICAL TECHNOLOGY ‘The current, at a power, half of the maximum power developed at resonant frequency, is 1 v Tr 1, where J, is the series resonant current i.e, FE According to the definition of bandwidth, or or According to Fig. 48 1 1 wl--R u-.-R ae . oe Oe Adding equations (i) and (ii), we get (, sopt-2(242] =0 My, Wg or (osopt-2(222) 29 Since Ww, + Org #0, Gyn, =i 1 #@g #0, ying = Te. 1 Again ©,” = 7G" (a, is the angular frequency at resonant condition. | Subtracting equations (i) and (ii), we have Lay -«,)+ 2 (tat )-2e Wath; 1 (wg-u,) _ 2k or (oy 0) + gE (wg -u) _ 2R or (mg — 0) + yg. REE = or (a -0) = 2 or eat Wo {w, is the angular frequency at resonant condition} i or 2-01 = Wo XT R or 2 -@y = olf © 4 ae AG, CIRCUITS 91 or f-h=k b> w= 2nfl Q or Bandwidth at series resonant condition = £. Sa Be at et LE w,-0, RIEL R RJLC RYC Q-factor of a resonant series circuit: The Q-factor of an R-L-C series circuit can be defined in the following different ways : (i) Q-factor is defined as the voltage magnification in the circuit at the time of resonance. Vv Since at resonance current is maximum i¢., J, = R ‘the voltage across either coil or capaci- tor = I, X,, or I, X,, and supply voltage, V = J, R. Vy, _ 1X, Xp _o,L _ Reactance ification = 4 = “ke 2 Sh 2 Se | Voltage magnification = = TR” R ” R” Resistance Vey 1 x, _ Xe. Reactance i Vv IR R Resistance w,CR Q-factor = Suk hE tan .{B(a] or where 0 is the circuit power factor angle of the coil. (At resonance, circuit phase angle 0 = 0, and Q = tan 0 = 0) (ii) The Q-factor may also be defined as under : maximum stored energy * are Q-factor = 2x . in the circuit 1yyt i 2 oon ae = on BY - Repth ee (te - ret PRT, ROVE) rR R\ a CR Tt 1 1 But t fi he = ut resonant frequency, f,= =—7ee or 2a, TE Putting this value in eqn. 8(a), we get Qtactor = MEL, te LE AB(6)) In series resonance, higher quality factori.e., Q-factor means higher voltage magnification as well as higher selectivity of the tuning coil. Example 32. A resistance 12 Q, an inductance of 0.15 H and a capacitance of 100 uF are connected in series across a 100 V, 50 Hz supply. Calculate : (i) The current. (ii) The phase difference between current and the supply voltage. (iii) Power consumed. Draw the vector diagram of supply voltage and the line current, Solution. Given : R=12Q,L=0.15 HorX, = 2nfL = Qnx 60x 0,15 =47.19 C = 100 pF = 100 x 10°F 1 or X, --L- ©" 2xfC 2nx 50x 100x 10° =3189 8 ELECTRICAL TECHNOLOGY 100 V, 50 Hz. (a) R-L-C cirenit (b) Vector/phasor diagram Fig. 49 {i) The current, I: Ze iF +(X,- Xe" = j12?+47.1-31.8" =19.439 Current, Is v. 100 = 65.15 A. (Ans.) Z 19.43 (ii) Phase difference, ¢: e=cos Bor tan? %u-Ac] . 138.8 . cos” > alo tan“! 2-5 | = 52° (lag) Hence current lags supply voltage by 52°. any (iii) Power consumed, P P=Vicos¢ = 100 « 5.15 « cos 52° = 371.1 W. (Ans.) Fig. 49 (a), (6) show the circuit and vector/phasor diagrams respectively. Example 33. For the circuit shown in Fig. 50 find the values of (i) current I, (ii) V, and V,and Cid pf. Draw the vector diagram. (Bangalore University) Solution. Refer Fig. 50 109 0.05H 2022 O1H = 50)F 200 V, 50 Hz: (a) Series circuit Fig. 50 AC. CIRCUITS 93 R=10+20=302 L=0,05 +01 =015H X= 2nfL = 2n x 50 x 0.16 = 47.12 1 1 X,2——* 263.72 C° Saf” On x 50x 10° x 60 X= 47.1-63.7=- 16.69 z= {R? +X? = fan? +(- 16.6" =3439 . Vv _ 200 a Te Zo gg 7 SBIA. (Ans,) ii) X,, = 2n x 50 x 0.05 = 15.79 Z,= ¥10"+ 15.77 21869 V, = 12, = 6.83 x 18.6 = 108.4 V. (Ans.) @, = cos“ (10/18. 57.6" (lag) Xyg = 2n x 50 x 0. 1.42 X=314-68.7 =- 3239 2,5 20" +(-32,9) =38Q V, = 12, = 5.83 x 38 = 221.5 V. (Ans.) 6, = cos“ (20/38) = 58.2° (lead) R_ 30 (ili) Combined p.f. =cos = 7 = 39 = 0.875 (lead) Vector diagram is shown in Fig. 50 (b). Example 34. For the circuit shown in Fig. 51. Calculate : () Current ; i) Voltage drops V,, Vz and V; : (tii) Power absorbed by each importance ; (iv) Total power absorbed by the circuit. Take voltage vector along the reference axis. 6o 60 120 WO BO 100 20° Fig. 51 (Pune University) Solution. Z, = (6 +j6) Qj Z, = (12-16) 2; Z, = (8 +/0) Z = 2, + Zq4 Zq = (8 +j6) + (12 -j16) + (8 + /0) = (28-10) 2 Taking Vs V 20° = 200.20" = (200 +j0) eve 100 200K28+ f10) 10025 + j10) Z (28-f0) (28- fl0N28+ j10) (28)? + 10)" = ooo J) #3474113 94 ELECTRICAL TECHNOLOGY (@) Magnitude of current = (3.17)* + (1.13)? = 3,96.A. (Ans.) (i) V, = 1Z, = (3.17 + f1,13K8 + j6) = 3.17% 843.17 xj6+8 x j1.13 4/113 x6 = 25.86 + j19.02 + j9.04 - 6.78 = 18.58 + j28.06. (Ans.) Vo = [Bq = (8.27 + f1.18X12 — 16) = 38.04 - j50.72 + /13.56 + 18.08 = 56.12 - j37.16. (Ans.) Vy = [y= (3.17 + f1.13X8 + j0) = 25.36 + j9.04. (Ans.) [Vs V, + V, + Vy = (18.58 + j28.06) + (56.12 ~ j37.16) + (25.36 + j9.04) = 100 + j0 (check)] Example 35. Fig. 52 shows a circuit connected to.a 230 V, 50 Hz supply. Determine the follow- ing: (i) Current drawn (ii) Voltages V, and Vy (iii) Power factor. Draw also the phasor diagram. 180 0.048 H 12Q 0.012H 120 pF 230 V, 50 Hz Fig. 52 (Pune University) Solution. Refer Fig. 52. Given : R,=18.Q,L, = 0.048 H; Ry = 129, L, = 0.012 H;C = 120 pF = 120 x 10 P. KX, = 2nfL, = 2m x 60 x 0.048 = 15.08 Q | X,, = 2nfL, = In x 50 x 0.012 = 3.772 1 1 — FF 28.59 2 “QnfC On x 60 x 120 10% Impedance, Z, = R, +jX,, = 18 +/16.08 = 23.48 £39.96" Xa Impedance, Z, = Ry +jX,9—iXp = 12 + j3.77 - j26.63 = 12 -j22.76 = 25.73 Z- 62.2" Totalimpedance, Z =Z, + Z, = (18 + 15.08) + (12 - 22.76) = 30 -/7.68 = 30.97 2-14.36" ‘Current drawn, I: Taking supply voltage as reference vector, ¥ = V0° = 2300°. 23020° —— 28020" _ 7.43 214.86". (Ans. soerz- ae 7 A“ , >7_¥ Current, foee= z AC. CIRCUITS 95 Voltages V, and V,: V,= 174.46 Voltage, ¥; = 7Z, =7.43.214.36" x 23.48 239.96" = 7.43 x 23.48 £(14.36" + 39.96") = 174.46 254.32" V. (Ans.) Voltage Vp = [Z, = 7.43 £14.36° x 25.73 2- 62.2° = 7.43 x 26.73 £(14.36° — 62.2") = 191.2 2-47.84", (Ans.) Phase angle between supply voltage and current ie, Vand J, o = 14.36° (lead) Power factor, cos $: cos @ = cos (14.36°) = 0.9687 (leading). (Ans.) V,=191.2V [= ¥, + V, = 174.46 264.32" Fig. 53 + 191.2 2—47.84° = 230 20° (check)] Resonance, Q-Factor and Bandwidth Example 36. A circuit consisting of a coil having an inductance of 0.25 H and a resistance of 3.Qis arranged in series with a capacitor of capacitance 20 pF. Calculate at what frequency resonance will take place and current flow ifan alternating valtage of 40 V at the resonant frequency is applied to the circuit. Find also the voltage across the capacitor. Solution. Resonant frequency, f, = ate iL in henry, C in farad] = 1 = 71.2 Hz. (Ans.) any0.25 x 20x 10° vo At the resonant frequency, J, (Or Imac) = -2 = 13,33 A, (Ans.) Voltage across the capacitor, V, = IX, 13.33 = +— (where C is in farad) one, Where is in fara 13.33 “Selling niet = 1489.8 V. (Ans.) Example 37. A coil of inductance 0.64 H and resistance 40 Q is connected in series with a capacitor of capacitance 12 wF. Estimate : (i) The frequency at which resonance will occur. (ii) The voltage across the coil and capacitor, respectively and also the supply voltage when a current of 1.5 A at the resonant frequency is flowing. (iii) The three voltages in (ii) with a current of 1.5.A flowing at a frequency of 50 Hz. _ 1 sag Solution. (i) he Rie UW is in henry, C is in farad] 1 * gpjoeixiaxi0™ Giniaeio# = 67.4 Hz. (Ans.) (ii) At resonance the supply voltage = IR = 1.5 x 40 = 60 V. (Ans.) 6 ELECTRICAL TECHNOLOGY Voltage across the coil = Hr +X? = 1.5 {40% (Qn x 67.40.64)? = 351.4 V. (Ans.) Voltage across the capacitor = IX, 1 =Ix aye (Cin uF] 15 = = 46.6 V. (Ans. | Qn x 67.4% 12% 10" , (iii) At 50 Hz: Voltage across the coil =l Ve? +X,2 = 1.5 (40? + (2n x 60x 0.64)? = 307.5 V. (Ans,) Voltage across the capacitor == IX, = 15 ____ - gas v. (Ans) 2n x 60x 12x 10 Voltage across the entire circuit, Vs NR +(x, - Xo =15 \ (40)? + {x x 50 x 0.64) } = 1.5 yl40)? +(- 64.2)" = 113.5 V. (Ans) Example 38. A coil having an inductance of 50 mH and resistance 10 Q is connected in series with a 25 F capacitor across a 200 V A.C. supply. Calculate : (i) Resonance frequency of the circuit ; (ii) Current flowing at resonance ; (iit) Value of Q by using different data. (Bombay University) 1 1 Solution. (i) = = 142.3 Hz. (Ans.) f* SeEE ~ aa f60%107 x 25x10" ai) Inas * 4 = 20.4, (Ans. aan 1 [L 50x 10 iii) Q=5 & 70 Voscio® 7447 (Ans Example 39. A series R-L-C series circuit consists of R = 800 Q, L = 80 mH and C = 8 pico- farad. The applied voltage across the circuit is 100 V. Determine : (i) Resonant frequency of the circuit. (ii) Q-factor of the circuit at the resonant frequency, (iii) Bandwidth of the resonant circuit, (iv) Frequencies at which the half power points occur. (v) Bandwidth of the circuit. (Delhi University) Solution. Given : R = 800 Q; L = 80 mH = 0.08 H; C=8 = 10°? F ; V = 100 volts. (i) Resonant frequency of the circuit, f,: 1 1 he = = 198.94 kHz. (Ans.) 2nJLC 2ny0.08 x8 x 10"? AC. CROUITS 7 ii) Q-factor at the resonant frequency : 1 jE _ 1 | 0.08 (Q-factor) csreacttequney = if. = Bao Pee 7125. (Ans, Gif) Bandwidth of the resonant circuit ; B,,; R 800 3,,°—=-——— / he Se Seco gg = ISLS He. (Ans) Also, a,-&. a = 1.5915 kHz =1591.5 Hz ...a8 above (iv) Frequencies at which the half power points oreur OW? 500 ae (kHz) = 198.14 kHz. (Ans.) R hho —s 198.94 (KHz) ~ 0 08 Jooo 800 fh= Ot ar H = 198.94 + Goa as* “yaop = 199.74 ki. (Ans.) (v) Bandwidth of then resonant circuit : Bandwidth = f, — f, = 199.74 — 198.14 = 1.6 kHz, (Ans,) Example 40. A series R-L-C circuit consists of R = 20 9, L = 20 mH and C = 0.5 uF. If the circuit is connected to a 20 V variable frequency supply calculate the following : (i) Resonant frequency f,. (ii) Resonance circuit Q-factor using L/C ratio. (iit) Half-power bandwidth, using f,and Q-factor. (iv) Half-power bandwidth using the general formula for any bandwidth. (v) Half-power bandwidth using the given component values. (vi) Maximum power dissipated at f,. Welhi University) Solution. Given : R = 20 9; L = 20 mH = 0.02H; C = 0.6 x 10°F; V = 20 volts. (i) Resonant frequency, f, = ie 1 = onjooaxo belo? = 1591 Hz. (Ans.) os j_0.02 02 (ii) Q-factor = ife-2 20 05x10" =10. (Ans.) 1591 (it) Halt-power bandwidth (using = ratio}, By, = x =o aot = 168.1Hz, (Ans) (iv) Half-power bandwidth (using the general formula), 591 tan 45° Byy = ee — - Mota tans score, (Ans) (Qresensnee 40 (+ At half power points, Q = tan 6 = tan 45° = 1) (v) Half-power bandwidth (using component values), R 20 By = == 5. he” Ser" Teng og 715015. (Ans) (vi) Maximum power dissipated at f_, y B= iir-(¥ ar oa _ sow. (Ans.) 98 ELECTRICAL TECHNOLOGY 4.6. A.C, Parallel Circuits 4.6.1. Introduction Now-a-days, owing to multiple system of transmission and distribution, we come across par- allel circuits (e., impedances joined in parallel) more often. Practically all lighting and power cir- cuits are constant voltage circuits with the loads connected in parallel. In a parallel A.C. circuit (like parallel D.C. circuit) the voltage is the same across each branch. 4.6.2. Methods for solving A.C. parallel circuits The following three methods are available to solve such circuits : 1, Phasor or vector method. 2, Admittance method 3. Vector algebra (symbolic method or j-method) 1. Vector or phasor method: Consider a parallel cireuit consisting of two branches of impedances Z,(R,, L) and ZR, C) respectively, and connected in parallel across an alternating voltage V volts (r.m.s.), as shown in Fig. 54 (a). Since the two branches are connected in parallel therefore, the voltage across each branch is the same and equal to supply voltage V but currents through them will be different. v (a) Circuit diagram (b) Phasor diagram Fig. 54. Single-phase parallel cireuit—Phasor method, Branch-1 Impedance, Z, = (Ry? + X,? Current, ie z 3 ‘b Power factor, cos = 2 or %= cow), 1 1 Current, /, lags behind the applied voltage by 0,. Branch-2 Impedance, By JR +X: Vv Current, has: eh Power factor, 608 by = = or = cos’ (2) A.C. CIRCUITS Current J, leads V by 6, (Fig. 54 (6)] Resultant current J, which is phasor sum of J, and J,, can be determined either by using parallelogram law of phasors, as shown in Fig. 54 (b) or by resolving branch currents J, and J, along X-axis and Y-axis and then determining the resultant of these components analytically. . Component of resultant current / along X-axis = Sum of components of branch currents J, and /, along X-axis or Teas 6 = 1, cos 4, + I, 008 >, Similarly, component of resultant current / along Y-axis = Sum of components of branch currents 7, and J, along Y-axis or fsing=— 1, sino, + J, sind, * T= fF, 008 6) + Fy cos Oy)" + (ly sin bq — 1, sin OF" AB) and tang= /28ingz—Asing | Y-component Aa] 7, £08 0; + J c05q X-component If tan @ is +ve, then current J will lead the applied voltage V, if ¢ is -ve current J will lag behind the applied voltage V. Power factor of the whole circuit is given by, cos 6 = 15989) +12 con be. Xcomponent a) 2. Admittance method : Admittance (denoted by symbol ¥) ofa circuit is defined as the reciprocal of its impedance. _ EMS. amperes rms. volts ‘The unit of admittance is siemens (S). The old unit was mho (0). As the impedanceZ of a circuit has two components R and.X (See Fig. 55), similarly, shown in Fig. 56, admittance Y also has two components G (conductance-X-component) and B (susceptance-¥- ‘omponent). 11 Yoory G (Conductance) o @ § z a Obviously, Similarly, 2 Admittance, ¥= /G? +B? just as Z=JR+ x? The units of G, B and Y are in Siemens. Here, we shall consider capacitive susceptance as +ve ad inductive capacitance as —ve. 100 ELECTRICAL TECHNOLOGY Application of admittance method in solution of single-phase parallel circuits: Refer Fig. 57. Determine conductance and susceptance of individual branches from the rela- tions = 5 naae 3 Taking B as +ve ifX is capacitive and as -ve ifX is induc- tive. Let the conductances of the three branches of circuit shown in Fig. 57 be G,, G, and G, respectively and susceptances be B,, B, and B, respectively. Total conductance is found by merely adding the conductances of three branches, Similarly, total susceptance is found by algebraically adding the individual susceptances of different branches. Total conductance, G=G,+G,+G, and, total susceptance, B=B, +B, +B, Fig. 57. Admittance method. Total admittance = ¥ = ¥G" + B? (10) ‘Total current, i=VY (12) Power factor, cos g = ¢ ~A12) 3. Complex or Pkasor algebra: Consider the paralle! circuit shown in Fig. 58 in which two impedances Z, and Z,, being in parallel, have the same potential difference across them vo Fig. 89 7_V¥ 7 _¥ Now, i= Ig= = low, 1 Zz, and J Zz and current To+h- 44 44 “¥(F+2)-78- %)-07 (3) 2, ty where ¥ (= total admittance) = ¥, + ¥ Add) It may be noted that theadmitiances are added in parallel branches, whereas impedances are added for branches in series. AC. CIRCUITS 101 It is most important to remember that admittances and impedances being complex quantities must be added in complex form. Let us now consider the two parallel branches shown in Fig. 59, we have F-t-_1__ R,- JX, 2 Ry+ iX_ CR, + SLR, ~ JX) ars Re+X,? R2+X? i where G,= qi: ...Conductance of upper branch, iL x, B,=-—-_, 1 Re+X2 ..Susceptance of upper branch. os ae 1 (Ry + Xp) Similarly, hp == 2 ot s °°" Re he U~ KoWR, + Me) + IX, = R OM - “ wheat Beek? Te ae Ot Total admittance ¥ = ¥, + ¥, =(G-jB,)+(G, +jB,) =(G, + G,)-j(B, -B,) = G -jB ¥ = y(G, +G,* +(B, - BP? (15) — an-t | Bi - Be and O= tan [a2 ; zs af 16) For admittance the polar form is: ¥ = ¥.29°, where 6 is as given above Totaleurrent =F =V¥;1,=¥ ¥,and1,=V¥% If ¥ =V20° and F = ¥26 then ra =V¥ =Vz0" x ¥2o=VY co Ingeneral,ie. VY =Véa and ¥ = Y2B, then TeV ¥ =Véax¥2p=V¥da+§) 17) Thus, it is warth noting that when vector algebra is multiplied by admittance either in eom- plex (rectangular) or polar form, the result is vector current in its proper phase relationship with respect to.the voltage, irrespective of the axis to which the voltage may have been referred to. Example 41, A resistance of 60 Q, an inductance of 0.18 H and a capacitance of 120 pF are connected in parallel across a 100 V, 50 Hz supply. Calculate : (8) Current in each path. (ii) Resultant current. (iii) Phase angle between the resultant current and the supply voltage. (iv) Power factor of the circuit. 102 ELECTRICAL TECHNOLOGY Solution. Given : R = 60; L = 0.18 H; C = 120 pF = 120 x 10°F, V'= 100 volts, 50 Hz. Inductance reactance, X, = 2/L = 2n x 60 x 0.18 = 56.55 ©, and . 1 1 nce reactance, —————___, = 26.532 sapacita ee, Xe 9G” bax 80x 120%10 (i) Current in each path: Current through reactance, l 1. = 1.67 A in phase with voltage V. (Ans.) Current through inductance, v 100 = 177 A lagging behind voltage V by 90". (Ans.) X, 56.65 400 V,50 Hz hat77a (a) Circut diagram (0) Phasor diagram Fig. 60 Current through capacitance, + 1% S.TTAL the voltage V by 90°. (Ans.) b=, 36.53 = leading the voltage V by 90°. The circuit and phasor diagrams are shown in Fig. 60 (a) and (b) respectively. (ii) Resultant current, I: Resultant current, 7 = JI)? + Uy - hy? = 67" +0.77- 1.77? =A. (Ans) (ii) Phase angle between the resultant current and the supply voltage, $: $= tans! (2-4) tan! (3) = 50.14" flead). (Ans) A . (iv) Power factor of the circuit, cos @: cos ¢ = cos 50.14° = 0.641 (lead). (Ans.) Example 42. Determine the r.m.s. value of current in each branch and total current of the circuit shown in Fig. 61. Draw the phasor diagram. (PTU, Jan. 2000)

You might also like